Wikipedia:Reference desk/Humanities: Difference between revisions

From Wikipedia, the free encyclopedia
Content deleted Content added
m Reverted edits by F (talk) to last version by Sam Korn
Line 868: Line 868:


Why do American legislators insist on giving their bills ridiculous acronyms? Looking through a couple of the relevant categories, I find [[USA PATRIOT]], [[PROTECT Act|PROTECT]], [[RAVE Act|RAVE]], [[BALANCE Act|BALANCE]], [[CARE Act|CARE]], [[CALM act|CALM]], [[COPE Act|COPE]], [[REAL Act|REAL]], [[SAFE Act|SAFE (twice!)]], [[SKILLS Act|SKILLS]], and [[USA Tax|USA]], and I could find many more if I looked through [http://thomas.loc.gov/ THOMAS]. Is there some historical reason? Are there any other countries with similarly absurd practices? --[[User:superiority|superioridad]] <sup>([[User talk:superiority|discusiĆ³n]])</sup> 11:03, 19 April 2009 (UTC)
Why do American legislators insist on giving their bills ridiculous acronyms? Looking through a couple of the relevant categories, I find [[USA PATRIOT]], [[PROTECT Act|PROTECT]], [[RAVE Act|RAVE]], [[BALANCE Act|BALANCE]], [[CARE Act|CARE]], [[CALM act|CALM]], [[COPE Act|COPE]], [[REAL Act|REAL]], [[SAFE Act|SAFE (twice!)]], [[SKILLS Act|SKILLS]], and [[USA Tax|USA]], and I could find many more if I looked through [http://thomas.loc.gov/ THOMAS]. Is there some historical reason? Are there any other countries with similarly absurd practices? --[[User:superiority|superioridad]] <sup>([[User talk:superiority|discusiĆ³n]])</sup> 11:03, 19 April 2009 (UTC)

== Opinion polls for [[European Parliament election, 2009 (Sweden)]]? ==

The [[Pirate Party]] got over 9000 new members recently, but does it stand any chance in the election? [[User:F|F]] ([[User talk:F|talk]]) 11:42, 19 April 2009 (UTC)

Revision as of 11:56, 19 April 2009

Welcome to the humanities section
of the Wikipedia reference desk.
Select a section:
Want a faster answer?

Main page: Help searching Wikipedia

Ā Ā Ā 

How can I get my question answered?

  • Select the section of the desk that best fits the general topic of your question (see the navigation column to the right).
  • Post your question to only one section, providing a short header that gives the topic of your question.
  • Type '~~~~' (that is, four tilde characters) at the end ā€“ this signs and dates your contribution so we know who wrote what and when.
  • Don't post personal contact information ā€“ it will be removed. Any answers will be provided here.
  • Please be as specific as possible, and include all relevant context ā€“ the usefulness of answers may depend on the context.
  • Note:
    • We don't answer (and may remove) questions that require medical diagnosis or legal advice.
    • We don't answer requests for opinions, predictions or debate.
    • We don't do your homework for you, though we'll help you past the stuck point.
    • We don't conduct original research or provide a free source of ideas, but we'll help you find information you need.



How do I answer a question?

Main page: Wikipedia:Reference desk/Guidelines

  • The best answers address the question directly, and back up facts with wikilinks and links to sources. Do not edit others' comments and do not give any medical or legal advice.
See also:


April 10

Next president of Kenya

Who will be the next president of Kenya? Will it be Odinga? Since how many mor years until Mwai Kibaki leaves office in Kenya? By 2010? 2011? Is that when Mwai Kibaki expects to step down from being a president? Who will take over Kibaki. Kibaki is 77 right now. By 2012 he'll be 81.--69.226.39.155 (talk) 01:23, 10 April 2009 (UTC)[reply]

Wikipedia is not a crystal ball. Editors should not (and hopefully will not) speculate about the future of the Kenyan Presidency. This discussion is better left for one of the thousands of discussion forums on the Internet. -- kainawā„¢ 03:59, 10 April 2009 (UTC)[reply]
  • No source said when Mwai Kibaki will leave. I don't know, I guess soon. That is because he's essentially old 77 years old, by December 2010-he'll be 79. Daniel Arap Moi left office when he was 78-that is December 2002. John Kufuor he is a ex-president of Ghana. He have been out of office since January 7, 2009, when he just turned 70 years old, and John Atta Mills took over him. I want to see if anybody knows when Mwai Kibaki will leave, I'm guessing Odinga will be next. I don't truly know. Maybe somebody else will be able to answer this question.--69.226.39.155 (talk) 04:29, 10 April 2009 (UTC)[reply]
As Kainaw has said, wikipedia is not a crystal ball. If no source has discussed when Mwai Kibaki will leave, then it definitely does not belong here Nil Einne (talk) 11:16, 10 April 2009 (UTC)[reply]

I only know what I read on Wikipedia, but as I understand it, the President of Kenya is chosen by election (every five years, last one in 2007). If you want to know if Kibaki will stand for reelection in 2012, you'd have to ask him. The same goes for Odinga or anyone else - if they haven't made any statements to the press about running in 2012, then you'd have to ask them personally. Whether or not Kibaki, Odinga or any other person stands for (re)election, it's ultimately up to the Kenyan voters to decide who to elect. According to our Kenyan presidential election, 2007, the previous election was bitterly contested, with some claiming the election results were flawed (see 2007ā€“2008 Kenyan crisis). With such an acrimonious result from the previous election, it is reasonable to conclude that any person running for the Kenyan presidency in 2012 will have difficulty establishing a clear majority of voters, so even if we did know who was running it would be difficult to predict the winner. -- 128.104.112.117 (talk) 15:22, 10 April 2009 (UTC)[reply]

Anon, you have written a slogan for a great T-shirt: I only know what I read on Wikipedia. Tempshill (talk) 19:46, 10 April 2009 (UTC)[reply]

Phoenicians trading with (Great) Zimbabwe?

And if so, when? See this diff, with later edits modifying and reverting the time of the supposed trade. See also this diff from the Zimbabwe article, changing Swahili people to Phoenicians as trading partners for Great Zimbabwe, which has survived several edits. I assume that both are vandalism, but ask here to make sure I don't mess up. --NorwegianBlueĀ talk 11:21, 10 April 2009 (UTC)[reply]

It must have been before the times of ridiculous inflation. The days of King Solomon's mines and all the gold you could eat have long gone. Sorry, this answer isn't remotely helpful.--KageTora (talk) 11:55, 10 April 2009 (UTC)[reply]
Yes it is mostly nonsense particularly as Phoenicia and great Zimbabwe's time is separated by about one thousand years. It is partly due to the myth that a civilisation cannot be entirely new and indigenous to a region and must be somehow 'imported' from another more advanced civilisation. Trade was a lot more widespread and distant in the ancient world than is usually imagined and the Phoenicians and people from that area of southern Africa might have had contact as the Phoenicians are said to have circumnavigated Africa but it was most likely brief contact. Also, even if it were true that descendants from Phoenicians lived somewhere in Africa for 1000 years they could hardly be regarded as Phoenicians any more. meltBanana 14:47, 10 April 2009 (UTC)[reply]
Whether it was willful vandalism or fanciful speculation, it doesn't belong in an encyclopedia. There is absolutely no evidence of any connection at all between the ancient Phoenicians and the more recent people of Great Zimbabwe. I have deleted the passage. Marco polo (talk) 18:50, 10 April 2009 (UTC)[reply]
Thanks. I did some detective work on the Zimbabwe article and related articles. In the introduction to Zimbabwe#History, there's a sentence that reads: "Around the early 10th century, trade developed with Phoenicians on the Indian Ocean coast, helping to develop Great Zimbabwe in the 11th century.". It's been sitting there for a long time (since May 22, 2008), through many hundreds of edits. It was recently changed to "Swahili people" by the same anon that introduced the supposed trade route to the Phoenicia article, but was promptly reverted! In the Great Zimbabwe article, there's a referenced statement by James Theodore Bent from 1891, that the builders of Great Zimbabwe must have been either Arabs or Phoenicians, but the article proceeds by citing referenced research concluding that the builders were African. No mention of a trade route. The Swahili people article has this to say about the matter: "Materials attributed to this network of trade were also found at Great Zimbabwe." In early versions of the Zimbabwe article, the history section referred to "Muslim merchants", not "Phoenicians". Here's the diff of the change from "Muslim Merchants to "Phoenicians", on May 22, 2008, by a registered editor. I'm changing it back to "Muslim merchants", and leaving a note on the article's and editor's talk pages.--NorwegianBlueĀ talk 21:13, 10 April 2009 (UTC)[reply]
The Phoenician connection I think is based on the early scholarship of Bent and others that saw similarities with the archeology of Phoenician colonies and hypothesized they had found a lost colony. This was soon superseded by less fanciful scholarship of indigenous builders although the Phoenicians were relegated to helping by being the main traders. The theory has remained current because people like epic founding myths and lost colonies over the slightly less thrilling explanation that local people did it, but also the colonial rulers did what they could to diminish the local African population and their earlier achievements. The east African coast has probably quite a varied but undocumented trading history and it is quite likely people from the Levant have traded there but probably not quite the influence sometimes accorded them. meltBanana 00:34, 11 April 2009 (UTC)[reply]
The "Phoenician" connection, like the longterm resistance to the idea that man evolved in Africa, not Asia, is a symptom of 19th-century European thinking that nothing genuinely new and important could have come from Africa, quite unlike the Romans, whose opinion was ex Africa semper aliquid novi a phrase often attributed to Pliny.--Wetman (talk) 07:25, 11 April 2009 (UTC)[reply]

is there a psychological reason for "code names" at companies?

First of all I carefully read the code name article. The relevant section is "Reasons for a project code name" (under the "commercial code names in the computer industry section"). However, of the five points identified, none of them are psychological (except for a 'fresh start' in the case of a failed project). My question is, is there a psychological aspect to giving a product a code name, ie one that would affect the team who is working on it. This is an aspect that is not mentioned in that article. I mean: if we assume the product will not be changing and already has its name, would there still be a psychological aspect (not pr or secrecy aspect) to giving the team a code name for it? For example, possibilities would be (wild guesses here) allowing the team to be more motivated or creative or in not to feel so much performance pressure on very famous products. Does anyone know if any of these are really a factor? 79.122.65.253 (talk) 15:00, 10 April 2009 (UTC)[reply]

Winston Churchill thought so. He wanted code names for military operations to be single words and inspirational (powerful-sounding) if possible. Sorry, this is from memory of his The Second World War (the one-volume abridged edition) and I don't have a page number to send you to. Tempshill (talk) 16:52, 10 April 2009 (UTC)[reply]
thank you, but if companies rely on the psychological aspect it certainly isn't because Winston Churchill thought there was something to it! There would be recent, corporate evidence.
Also: the iPhone was codenamed "p2" (for purple2, after the first phone apple collaborated on, code named 'purple'). If anyone knew about inspirational effects it would be Apple, but they didn't pick 'p2' because it is inspiring...
Of course, in this particular case the overweighing aspect is secrecy. However, beyond the secrecy, does the fact that you're working on p2 instead of the iPhone have salutary psychological effects? I mean salutary from the point of the view of performance in creating the product... 79.122.65.253 (talk) 17:18, 10 April 2009 (UTC)[reply]
I'd disagree that companies need "corporate evidence" to make decisions, but that's off topic. Probably your speculation about motivation and creativity are usually factors in the choosing of these code names, just as inspiration was part of Churchill's criteria. I doubt there have been academic studies of this, if that's what you're asking. Tempshill (talk) 19:45, 10 April 2009 (UTC)[reply]
It could if the "Purple" project was an astounding success. It could give the workers a sense of achievement that they worked on something well known enough that the general public knows about it. Or, in the reverse, if the original project was an abject failure, it could have an effect on employees if they are still working on "Project Pileofcrap." But this is OR, and like Tempshill said, I don't think there has been any research on this type of psychology. Livewireo (talk) 19:53, 10 April 2009 (UTC)[reply]
in this case by all accounts (including apple's, market and critical analysts' opinons, etc) "Purple" was a dismal failure both in terms of the end product designed and the reaction to it when it was brought to market. 79.122.65.253 (talk) ā€”Preceding undated comment added 21:14, 10 April 2009 (UTC).[reply]
A large reason for code names is that naming is non-trivial. Marketing must make sure that the name doesn't mean something undesireable in a foreign language. Legal has to make sure that the name doesn't infringe on copyrights. The name is usually tested with a panel of consumers against other potential names. All of this requires a significant amount of time. Rather than wait until the naming process is done or resort to calling the product, "that new thing the dev people cooked up," a company will assign a code name. Wikiant (talk) 22:06, 10 April 2009 (UTC)[reply]
Even if the ultimate name is known early on, I imagine it's more fun and less confusing for the engineers to use a name that sounds like a name rather than "PSC 1410". ā€”Tamfang (talk) 07:45, 11 April 2009 (UTC)[reply]

Certainly where I work high-profile/'secret' projects are code-named for privacy reasons. This way staff do not know that project Wensleydale is the company's secret project looking at reducing the workload by 20% or whatever. So from an internal perspective it can aid privacy and allow business-decisions to be investigated in a 'project' framework without being necessarily known about by many staff. ny156uk (talk) 22:49, 10 April 2009 (UTC)[reply]

But if a staff member hears about project Wensleydale and is curious what it's about then he will just ask someone. So how does that help ā€“ would the person who knows its content simply refuse to tell him? --Richardrj talk email 06:54, 11 April 2009 (UTC)[reply]
Basically, yes. The people on the project sign a confidentiality agreement whereby they are not allowed to discuss the project with anybody else (colleague, family or otherwise). Of course the purpose of the project is usually subject to rumours by people speculating what it is (or leaked info). ny156uk (talk) 07:15, 11 April 2009 (UTC)[reply]
Not entirely psychological, but it also impacts corporate espionage. It's harder for your competitors to find out about your new sprocket design if it's code named Snickerdoodle. ā€” The Hand That Feeds You:Bite 14:04, 13 April 2009 (UTC)[reply]

Heavens above! We will have to rapidly change our new products code name from Snickerdoodle; we thought it was our orginal!--79.71.217.59 (talk) 09:40, 17 April 2009 (UTC)[reply]

Poem

What's that poem in which the author says he/she hates to read because he/she feels more like the unreliable side characters than the protagonists? .froth. (talk) 18:31, 10 April 2009 (UTC)[reply]

'A Study of Reading Habits', from The Whitsun Weddings, by Philip Larkin. LANTZYTALK 11:54, 12 April 2009 (UTC)[reply]
You're good. --.froth. (talk) 05:37, 14 April 2009 (UTC)[reply]

Population of Egypt

I discovered that a section on population got removed from the Nile Delta article. Can anyone here confirm what is said there and restore it to the article with sources? Carcharoth (talk) 22:07, 10 April 2009 (UTC)[reply]

Done. Marco polo (talk) 22:26, 11 April 2009 (UTC)[reply]

is ninja blinding powder permanent?

I'm ashamed I can't find our article on ninja blinding powder, but was it intended to (did it) cause any permanent damange? 79.122.65.253 (talk) 23:24, 10 April 2009 (UTC)[reply]

It is not permanent according to this site [1]:
"Blinding Powder (external, immediate): Temporarily blinds the target unless they avoid getting it in their eyes and nose."
I cannot vouch for the site or the information // BL \\ (talk) 23:36, 10 April 2009 (UTC)[reply]
That sentence is contradictory. 'Unless' = 'if not', so 'if they do not avoid getting it in their eyes' it is temporarily blinding. However, the use of the word 'unless' implies another possible outcome, in which they do in fact avoid getting it in their eyes, and would also imply a harsher effect, such as permanent blindness. Of course, this was written by a teenager, so all can be forgiven.--KageTora (talk) 00:27, 11 April 2009 (UTC)[reply]
Huh? The implied alternate outcome (if the target avoids...) is that it does not temporarily blind. A genie, given the opportunity, would presumably read "does not temporarily blind" as "permanently blinds", but you're under no obligation to read it that way. ā€”Tamfang (talk) 07:50, 11 April 2009 (UTC)[reply]
Nope, Tamfang, 'unless' does not mean 'if', it means 'if not'.--KageTora (talk) 21:28, 11 April 2009 (UTC)[reply]
More precisely, it means but not if; that is, unlike a simple if or if not, it has something to say about both branches of the condition. If the target does not avoid, the powder temporarily blinds; AND: if the target avoids, something else (which may be nothing) happens, as you said yourself. We're disputing what that else is. (Am I feeding a troll here?) ā€”Tamfang (talk) 08:47, 12 April 2009 (UTC)[reply]

I found a clue on a web site: "The powder used in the old days was said to be so strong that it would make a opponent blind completely..." which as you can see from the phrasing is a pretty weak reference. Anyone able to do better? 79.122.65.253 (talk) 23:37, 10 April 2009 (UTC)[reply]

The WP article Metsubushi (ē›®ę½°ć—Ā ?) says the blinding can be either temporary or permanent depending on the nature of the specific powder. In this case, I can vouch for the site, but not for the information. // BL \\ (talk) 01:21, 11 April 2009 (UTC)[reply]

Baseball Team

What baseball team wears the hat with a capital A? Who is the guy who wears 34?68.148.145.190 (talk) 23:34, 10 April 2009 (UTC)[reply]

I think you are refering to Nick Adenhart of the Los Angeles Angels of Anaheim. Rmhermen (talk) 23:40, 10 April 2009 (UTC)[reply]
It could also be the A's but their number 34 seems to be retired in honor of Rollie Fingers. APL (talk) 01:32, 11 April 2009 (UTC)[reply]
Atlanta Braves also wear a capital A. Who then was a gentleman? (talk) 05:18, 11 April 2009 (UTC)[reply]
In Major League Baseball there are 3 teams with A's on their hats. The Atlanta Braves number 34 is Eric O'Flaherty. The Los Angeles Angels number 34 was worn by the recently deceased Nick Adenhart. The Oakland A's number 34 is (as noted) retired by the team, but it was most famously worn by Rollie Fingers. --Jayron32.talk.contribs 12:35, 11 April 2009 (UTC)[reply]


April 11

Painting: Victorian lady having tea

There exists this picture of a Victorian lady having tea. Is it a famous work? Is there a larger version available? ----Seans Potato Business 03:06, 11 April 2009 (UTC)[reply]

Going out on a limb, it's an Anglo-American? illustration (from a magazine?) of ca 1900. It is probably not a famous image.--Wetman (talk) 07:16, 11 April 2009 (UTC)[reply]
I would guess that it's one of the genteel quasi pin-up cards of Edwardian times, often depicting elegantly attired upper-class young ladies looking pensive. Not sure what the recognized term for that kind of thing is, or how you would track one specific item down without specific identifying information. In the United States during the same period, we had Gibson girl illustrations... AnonMoos (talk) 00:50, 12 April 2009 (UTC)[reply]
The artist was Harrison Fisher. And yes, the original shows more of the lady's dress. These folks [2] sell notecards with a print of the picture, they may be able to give you more info about it. Crypticfirefly (talk) 23:42, 12 April 2009 (UTC) I found out a bit more about it. These folks [3] say that the name of the picture is In Teacup Times and that it dates from 1909. They also allow you to download a more complete version of the image. Crypticfirefly (talk) 23:52, 12 April 2009 (UTC)[reply]

Is there a god of elections?

There is a god of love, there is a god of war. Is there a god of elections? Or at least a patron saint? ā€”Preceding unsigned comment added by 59.93.5.17 (talk) 08:00, 11 April 2009 (UTC)[reply]

Answers on the Language Desk meltBanana 12:50, 11 April 2009 (UTC)[reply]

removed question on Obama

This question has been removed as it is not appropriate for the Reference Dek. Please see the talk page. ā€”Preceding unsigned comment added by Mr.K. (talk ā€¢ contribs) 12:06, 11 April 2009 (UTC); preceding unsigned removal performed by Jayron32 (talk ā€¢ contribs) 12:30, 11 April 2009 (UTC) [reply]
The discussion about whether the Q should have been removed is here: Wikipedia_talk:Reference_desk#Q_removed:_.22Why_does_Barack_Obama_get_a_reception_as_if_he_was_Jesus_or_Hitler.3F.22. StuRat (talk) 15:56, 11 April 2009 (UTC)[reply]

Historical interest rates for CHF and USD

Could you help me where can I find historical base interest rates for the US dollar from 2001 up to the latest data? I would be very pleasured, if you could help me about the smae thing regarding the CHF. --Ksanyi (talk) 14:05, 11 April 2009 (UTC)[reply]

I can vouch for 'FRED' at the St. Louis Fed. It is a wonderfully easy resource to use. [4] This link goes straight to the section on interest rates. Remember that there are a lot of interest rates in an economy. T-Bill rates are good if you're looking for a risk-free 'cost of capital' type rate. It would probably serve as the best "Social discount rate" too. Prime rates are good if you're looking for what would face highly creditworthy people and businesses. I think you need to create an account to download the data, but it's quick and easy. I can't help specifically on the Swiss data.NByz (talk) 02:11, 12 April 2009 (UTC)[reply]

I thought to an article like this one, where there's given a base rate (here the interest rate paid after the federal funds). This is for my thesis. Can anybody help me to find data about swiss franc?

That article refers to the Fed Funds target range. This would be the range for interbank lending and (when credit markets are functioning well!) forms the basis for other rates in the economy. The Swiss equivalent seems to be the Repo Overnight Rate (or Repo Overnight Index). [5] This link seems to have data for the last couple years (the link "Current Interest Rates - Overview" at the bottom). Actually, that may be wrong. [6] This and [7] this link suggest that the swiss conduct monetary policy by setting a target range on the three-month LIBOR for CHF deposits. This isn't a comparable rate to the Fed Funds rate though.
Also, this [8] link has information on money market and pure discount bond rates.NByz (talk) 16:47, 12 April 2009 (UTC)[reply]

Any reason behind having CHF = Swiss Franc?96.53.149.117 (talk) 16:11, 14 April 2009 (UTC)[reply]

The "official" title of Switzerland is "Confoederatio Helvetica" - with German, French, Italian and Romansch speakers in the country, instead of picking out as single language for the "official" name, they used Latin instead. (See note 8 in the Switzerland article) -- 128.104.112.117 (talk) 16:46, 14 April 2009 (UTC)[reply]

Is there a religious costume for Protestants (for religious scholars and for common people)? Is there a special food eaten by Protestants (in special meals)? Excuse me if I have any grammar mistakes, my mother tongue isn't English. 19:28, 11 April 2009 (UTC) ā€”Preceding unsigned comment added by 89.138.52.171 (talk)

The article Clerical clothing has a section on clothing worn by some protestant ministers. Protestant lay people wear widely varying clothing, with a few denominations such as Amish wearing distinctive "plain" clothing. Some Mennonites also wear distinctive clothing. Mormons wear special Temple garments under their clothing, but many do not consider them be Christians, let alone Protestants. One "special meal" eaten by Christians is called the "Lord's Supper" or Eucharist. It consists of bread and wine, or either element alone, or bread and grape juice, and in some denominations merely symbolizes the Last Supper, but in other denominations is thought to be miraculously transformed into the blood and flesh of Jesus. Protestant Christians otherwise generally eat ordinary food of whatever nation they live in, without special religiously based dietary restrictions. Edison (talk) 19:46, 11 April 2009 (UTC)[reply]
I've noticed that (in the United States) many evangelical clergymen who have their own TV shows wear academic dress when haranguing their congregations, which has always seemed odd to me. Deor (talk) 20:05, 11 April 2009 (UTC)[reply]
I expect that it was actually our form of clerical dress (choir dress). Academic dress is pretty rarely used in the U.S. even by academics and we aren't good at preserving such distinctions (tuxedo vs. morning dress, etc.) The only two televangelists so dressed that I noticed while scanning Google were D. James Kennedy (wearing a blue robe whille his academic robe would be violet for New York University) and Robert Schuller whose doctorates all appear to be honorary. Rmhermen (talk) 21:18, 11 April 2009 (UTC)[reply]
While a few, mostly small Protestant groups do have distinctive clothing for common (lay) people, most do not. They wear the same clothing as secular (nonreligious) people and members of other religious groups in their countries. Likewise, most Protestants eat the same food as secular people and members of other religious groups in their countries. It is more common for Protestant clerics to wear distinctive clothing, but there are also many Protestant clerics and scholars, at least in the United States, who do not wear distinctive clothing. That is, they wear the same kinds of clothes as people with a similar class background in their country. Marco polo (talk) 21:49, 11 April 2009 (UTC)[reply]
Just to clarify Marco polo's comment, it is more common for a Protestant cleric to wear religious clothing than a Protestant layman. It is more common for a Catholic cleric (or, I think an Orthodox one) to wear religious clothing than a Protestant cleric. Rmhermen (talk) 22:00, 11 April 2009 (UTC)[reply]
To clarify about the shared meal: The belief in Transubstantiation (the miraculous transformation of the substance of bread and wine into Christ's body and blood) is not a Protestant belief; it is distinctive to Roman Catholic Christians. The Protestant view of the Eucharist ranges from Martin Luther's view, sometimes described as consubstantiation or the Real Presence, that there is a real spiritual change in the elements, but not an alteration of physical substance, to the bare memorial interpretation which holds that the Eucharist is simply a shared meal, performed in obedience to Jesus' instruction 'Do this in remembrance of me'. The original Eucharist - the Last Supper - was probably a Passover Seder, and so many Christians use unleavened bread for celebrating the Eucharist.
As for clerical dress - this ranges from Lutheran and Anglican churches which closely follow Roman Catholic convention, through academic dress, choir dress, a combination of both ('hood and scarf'), plain sober black, and on to ordinary smart everyday clothes. AlexTiefling (talk) 13:54, 12 April 2009 (UTC)[reply]
The belief in Transubstantiation is not only held by the Roman Catholic Church. It is also held by other Sui iuris Churches in communion with Rome as well as the Eastern/Greek Orthodox and other (mostly Eastern) churches. --JoeTalkWork 01:00, 13 April 2009 (UTC)[reply]
And indeed by a not insignificant minority of High-Church Anglicans. [[Sam Korn]] (smoddy) 00:16, 18 April 2009 (UTC)[reply]
While there are few Protestant denominations that forbid or require eating specific foods, it's also true that denominations develop subcultures of their own in which certain foods may figure. Think of church lady jello salads and baked macaroni and cheese topped with bread crumbs, or lemon ice box pie, or Tic-Tac pie, easily identified as mid-Western U.S. Protestant dishes. - Nunh-huh 03:07, 13 April 2009 (UTC)[reply]
It's really impossible to answer the OP's question directly. "Protestant" pretty much means ANY non-Catholic western Christian sect which developed after the 16th century. It tends to cover the "mainline" protestant sects like Anglicanism and Baptist and Methodist and Lutheran and Presbyterian as well as other rather esoteric smaller sects like the Quakers and Seventh Day Adventist and Amish. Depending on who you ask, some rather latecomers to the Christian world, like Jehovah's Witness and Mormon and Christian Scientist also get lumped in to the "Protestant" umbrella. Other than 1) originating in Western Europe or North America and 2) Being Christian and 3) Not following the Pope there are NO real other commonalities between ALL of the various Christian groups that get labeled "protestant". With regard to the vestments, some denominations where clerical robes similar to Catholic Priests do; some wear "academic" or "choir" style robes, some wear a nice suit and tie. It REALLY varies widely between denominations, and sometimes from congregation to congregation WITHIN one denomination. --Jayron32.talk.contribs 04:54, 13 April 2009 (UTC)[reply]


April 12

Rodearmel v. Clinton

Hello! I'm interested in getting some information about the case Rodearmel v. Clinton. The article Saxbe fix mentions the case in its "21st century" section, and the lawsuit basically claims that Hillary Clinton can't legally serve as the Secretary of State.

This Fox News video clip on Youtube talks about the lawsuit, and the reporter said about 1:55 into the video that Clinton had 60 days to respond to the lawsuit. However, 60 days have passed, and according to this website, the only filing that could resemble a response was a "Notice of Appearance by Jeffrey Michael Smith on behalf of all defendants".

Therefore, was this "Notice of Appearance" supposed to be Clinton's actual response to the lawsuit? If so, it seems like the reporter on Fox kept the viewers in a 60-day suspense for nothing. I hope I'm not misinterpreting any of the information I'm looking at...

In addition, I am thinking about starting a new article about this case, but I'm worried that it might not meet the Wikipedia:Notability policy. Any thoughts? ā€”Preceding unsigned comment added by Edge3 (talk ā€¢ contribs) 02:32, 12 April 2009 (UTC)[reply]

Basically the Saxbe Fix is well-accepted as fullfilling the Framers' intent in writing the ineligibility clause, and the folks opposing H. Clinton's appointment on ineligibility clause grounds are about as likely to succeed as the birthers claiming Obama is ineligible to be president on citizenship grounds. The Clinton case is appropriately mentioned in the Saxbe Fix article, but isn't worth writing a separate article about. 66.127.52.118 (talk) 09:54, 12 April 2009 (UTC)[reply]
That may be true, but Clinton is still required to respond within 60 days of the legel filing, right? Therefore, has she decided not to respond to the lawsuit at all, or is the "Notice of Appearance" essentially her response?--Edge3 (talk) 14:55, 12 April 2009 (UTC)[reply]
Actually, now that I think about it, could the 60 days have referred to 60 business days?--Edge3 (talk) 04:13, 13 April 2009 (UTC)[reply]

Living long as possible

What is with the current obsession to live as long as possible? Lust for life, or fear of death? Desire to be fit, or obsession over looks?

My personal motto is that if my body isn't completely destroyed by the time I die, I'd consider it a waste of resources. I'd rather live 70 years and die in a painful two week span then live til I'm 90 and spend the last 20 years in debilitating chronic pain and dementia. Not to mention 70 years of enjoyment > 90 years of boredom.

--Pieppiep (talk) 11:04, 12 April 2009 (UTC)[reply]

Why do you suppose that the obsession is "current"? The desire for long life (and by extension immortality) is by no means a recent fad. I think most people would claim to agree with you about preferring a shorter, more active life to a longer, more decrepit one, but how many people actually put this attitude into practice? For many people (perhaps for most), 'fitness' is relatively unimportant. What is important is a relative absence of pain and inconvenience.
Also, I don't see how obsession over looks would contribute to a desire for a longer life. If anything, it would dispose you to the opposite: live fast, die young, and leave a beautiful corpse. LANTZYTALK 11:51, 12 April 2009 (UTC)[reply]
It's of note that the young often have different views towards what they'd like when they are old than when they are actually themselves old. Similarly lots of able-bodied people have extreme views about whether they'd want to continue living if they were suddenly rendered disabled, but in practice most who end up so disabled suddenly discover that they'd like to keep on living anyway. Anyway, the problem with your scheme is that you probably aren't as good an estimate of what kinds of activities will make you live until age 70 anyway. If you consciously try to "destroy" your body by age 70, e.g. adopt a pattern of habits you think will incrementally lead to a total expenditure of bodily health by age 70, you might only make it to 55 and find yourself feeling a little gypped. (I knowā€”any age decades in the future sounds "old", but they arrive a lot quicker than you think.) --98.217.14.211 (talk) 12:29, 13 April 2009 (UTC)[reply]
Further, many people would rather you didn't destroy your body (through whatever means) before you die. Organ donation is a rather important thing. ā€” The Hand That Feeds You:Bite 14:16, 13 April 2009 (UTC)[reply]
Pieppiep: your 70/90 assumption is wrong. That's not the choice you face. The anon is quite right about this. Life extension advocates call this idea the 'Tithonus error'; see 'The Tithonus option is not an option' and 'Combating the Tithonus Error: What Works?'

shemale

What does Islam say about Shemale? Is there a punishment for a woman having a penis?

That's an interesting way to phrase it. You might take a look at mukhannathun and khanith, both concepts within Islam relating to transsexuality. LANTZYTALK 13:57, 12 April 2009 (UTC)[reply]
You may also wish to avoid referring to transgendered or transsexual people as 'shemale'; it is a term coined by the pornography industry, and is regarded as highly offensive and demeaning. The Wikipedia article 'Shemale' includes a fair range of well-sourced criticisms of the term from commentators including transpeople themselves, and others. AlexTiefling (talk) 14:06, 12 April 2009 (UTC)[reply]
You'll need to qualify what you're talking about, first. Are you discussing individuals undergoing gender reassignment, or are you talking about people born intersexed? Further, there's more than one branch of Islam, just like there's not a single Christianity that could answer this question. ā€” The Hand That Feeds You:Bite 14:18, 13 April 2009 (UTC)[reply]
Eh? Isn't shemale an Arabic word for 'north'? ā€”Tamfang (talk) 03:45, 14 April 2009 (UTC) [reply]

How much Christians were killed during the Holocaust?

I read the Holocaust article. But there is nothing about killed Christians. Is it true, that only Non-Christian-people were killed by the Nazis? --Manuel-aa5 (talk) 15:43, 12 April 2009 (UTC)[reply]

The Nazis targetted and killed several categories of people. The Jews were the most numerous and their genocide is the most well remembered. Groups that were not Jewish would have had some secular members, some nominal Christians, and some committed Christians. For example, the Nazis did not like gay people, degenerate artists, the Roma (Gypsies), trades unionists, and so on. Nazi concentration camp badges offers a handy shortcut to categories of hate. BrainyBabe (talk) 16:14, 12 April 2009 (UTC)[reply]
While I'm not an expert on the holocaust, I'm pretty sure the Nazis used a very loose definition of Jewish people and Judaism was only one of their considerations. So it's likely some of the people who were killed as 'Jews' were Christians as well. Nil Einne (talk) 10:50, 14 April 2009 (UTC)[reply]
The Nazi party was ruthless in its persecution of any dissentors, regardless of religion. Leaders and members of the Protestant Confessing Church were persecuted and sometimes killed by the Nazis - see Dietrich Bonhoeffer and Martin Niemƶller, for example. Catholics also died in the concentration camps - see Edith Stein and Ɖlise Rivet. Gandalf61 (talk) 17:11, 12 April 2009 (UTC)[reply]
The reason that the many Christian and other non-Jewish victims of the Germans are not mentioned in the Holocaust article is that the Holocaust is a term that is most usefully and specifically descriptive of the Germans' anti-Jewish genocide. As the article states, in the strictest definition of "the Holocaust" the term is applied solely to Jewish victims, though not everyone uses the term so specifically. - Nunh-huh 02:57, 13 April 2009 (UTC)[reply]
There is some more info at Holocaust victims #Religious targets. ā€” Kpalion(talk) 13:40, 13 April 2009 (UTC)[reply]
Two million Catholic Polish were killed in the Shoah. It wasn't relevant to the Nazi orchestrators that they were Catholic Christians, only that they were Polish/in the way. If you want a good story of a Christian who died in a concentration camp check out St. Maximilian Kolbe. --JoeTalkWork 23:31, 13 April 2009 (UTC)[reply]
While "Holocaust" is (too) often used loosely, "Shoah" usually isn't. If someone was not Jewish, he wasn't "killed in the Shoah". Being killed in a concentration camp is not equivalent to being killed "in the Shoah". - Nunh-huh 01:53, 14 April 2009 (UTC)[reply]

Is it legal - Not a legal advice, just a question.

When I go to work, I drive on a dark road, my question is, if a police car wants to stop me, is it legal to signal to the cop you notice him but that for my own safety I want to drive to the closest place where there are people around? or... can he force me to stop there (on the dark road). --190.49.107.126 (talk) 18:29, 12 April 2009 (UTC)[reply]

Asking whether or not it is legal to do something is asking for legal advice. If you want to know what specific law may be in place with respect to the time or distance you are permitted to travel having been requested to stop by a police officer, then someone here might be able to help you if you tell us what jurisdiction you are in. As for forcing you to stop, that is hard to do without a road block as O.J. Simpson proved to the world. (I am interested that you live somewhere where it is being in a dark place with police officers that alarms you.) What you will likely be dealing with as soon as you finally pull over is the level of irritation in the police officers that has risen with every passing minute of "chase" and what effect that might have on their attitude towards you or their ticketing response. There will also be a difference in their immediate response if you are a 150-cm, 40-kg, 17-year-old female or a 2-metre, 90-kg, 30-year-old male. // BL \\ (talk) 19:21, 12 April 2009 (UTC)[reply]

I am 19 year old female in Argentina, some girls have been raped by police officers and one murdered years ago. --190.49.107.126 (talk) 19:33, 12 April 2009 (UTC)[reply]

  • If I were in that situation, I'd put my hazards on and proceed cautiously to a well-lit gas station or something of that sort. I would also consider phoning the police on my cell phone to explain what I was doing, if that was something safe and possible to do. Crypticfirefly (talk) 23:24, 12 April 2009 (UTC)[reply]
That's reasonably good advice right up to the part about using a cellphone. Chances are that the cop hardworking police officer is going to be upset as a result of the delay; providing him an opportunity to also cite you under DWD ("Driving While Distracted") laws isn't such a great idea. Also, fiddling with anything out of sight will increase the stress on the officer (who might assume it is a gun), escalating a traffic stop to a full arrest. Nothing in your explanation is time-sensitive; find a place to pull over *then* carefully and clearly make the call if you feel it is important (such as if the policeman is in an unmarked car). ā€“Ā 74Ā  04:02, 13 April 2009 (UTC)[reply]
Any laws that prohibit using cell phones while driving do not apply if you are calling a police station or any other emergency services agency.--Edge3 (talk) 04:11, 13 April 2009 (UTC)[reply]
Care to reference that statement? Here is a counter-reference: NHTSA legislative survey. Besides, attempting to argue law with a police officer is a futile undertaking; at best (but not at all guaranteed) you will be able to have the charges dismissed in court. ā€“Ā 74Ā  04:33, 13 April 2009 (UTC)[reply]
According to this Illinois law, cell phone usage while driving is allowed while calling an emergency services agency.--Edge3 (talk) 04:49, 13 April 2009 (UTC)[reply]
Which is markedly different from "any", appears to be part of a "graduated licensing" program, still leaves open the question of whether or not a traffic stop is an "emergency", and is certainly no proof against a police officer who wants to write you another ticket. ā€“Ā 74Ā  05:02, 13 April 2009 (UTC)[reply]
The reason for calling the police station is irrelevant; it would not make sense for a jurisdiction to prohibit people from calling the police. However, you're right that this law clearly doesn't apply in Argentina. I'm going to ask around in Spanish Wikipedia.--Edge3 (talk) 05:08, 13 April 2009 (UTC)[reply]

I used to work for a company (in the UK) that had "vulnerable loads" - they had a special sign for drivers to hold up to the window if they were signalled to pull over by the police that read something like.

This vehicle is carrying a vulnerable load. The driver is instructed not to unlock doors or open windows. If required he/she will follow you to the nearest police station.

and on the reverse were instructions like

Vulnerable load instructions. When signalled to stop by the police you MUST obey, however beware of bogus police. Stop vehicle, keep engine running, do not open windows or doors. Display the other side of this notice through CLOSED window. If required display driving license, work record or tachograph disc through CLOSED window. If required to follow the police officer to a police station make certain it is a police station before stopping engine and/or unlocking doors

So in the UK it would appear you have to stop when instructed to but common sense allows the police to escort you to a police station for assessment rather than forcing you to exit your vehicle if you are not certain of the bona fides of the police officer. Exxolon (talk) 23:26, 12 April 2009 (UTC)[reply]

Just because the company believed they could get away with it doesn't mean there were no repercussions for the driverā€”the company's driver's license wasn't at stake, for instance, and they probably had a legal team to deal with any resulting charges. (As an aside, if a "vulnerable load" was so very valuable, stopping the vehicle to display a little sign advertising that fact is an open invitation for the would-be thief to draw his gun and threaten/shoot you through the "closed" window. (This, of course, does not apply to bulletproof glass windows.)) ā€“Ā 74Ā  04:24, 13 April 2009 (UTC)[reply]
This was an "official" police approved notice. A modified version of this procedure is still used - see [9] - as this is in the UK firearms are very rarely used in criminal activity. Exxolon (talk) 04:44, 13 April 2009 (UTC)[reply]
And here's a link to an example of such a card - [10]. Exxolon (talk) 04:47, 13 April 2009 (UTC)[reply]
This article says that you are allowed to continue driving until you arrive at a spot you think is safe.--Edge3 (talk) 03:58, 13 April 2009 (UTC)[reply]
That's for Jonesboro, though, and does not necessarily cover Argentina. // BL \\ (talk) 04:11, 13 April 2009 (UTC)[reply]
I can speak a little bit of Spanish; should I ask in the Spanish Wikipedia?--Edge3 (talk) 04:54, 13 April 2009 (UTC)[reply]
This is honestly, from memory, but I do recall when a number of incidents occurred in my area involving police impersonators, the police advised to signal to the officer that you see him/her but you are going to keep driving until you find a well-lit area. I would basically stick my hand out the window and just "wave ahead" to let him/her know to follow me until I find a gas station or whatever. Supposedly, the police are supposed to be understanding of this, however, I tend to believe I would end up with a highly irritated cop at my window. That said, I am happy to know that we do not have the same problem the OP does with this issue. ā€”Preceding unsigned comment added by 74.213.220.154 (talk) 08:06, 15 April 2009 (UTC)[reply]

Problem with Delhi Page

I cannot find any other page, so I am posting here.

The article on Delhi has a very high rating, but it completely fails to define exactly what Delhi is, a serious defect in the article. I have made a number of posts on the discussion page (Talk:Delhi) and on the Delhi workgroup page (Wikipedia_talk:WikiProject_Delhi), but have received pretty much no help.

The problem with the name "Delhi" is that it's unclear whether it's the name of a city, a territory or something else. My best guess at this point is that "Delhi" is a nickname for the National Capital Territory and is often used to mean "New Delhi." I have proposed adding the below sentence, but have received no replies, so I am reluctant to add it:

Properly speaking, Delhi is known as the National Capital Territory. There is no official administrative unit of India that is called Delhi.

Can anyone help clear this up? Wakablogger2 (talk) 19:42, 12 April 2009 (UTC)[reply]

The first sentence says Delhi is a "metropolis". What's wrong with that as a description? A city need not coincide with an administrative unit. ā€”Tamfang (talk) 20:59, 12 April 2009 (UTC)[reply]
What does it mean "a metropolis"? Is it a city? Is a metropolis a government unit? Is "Delhi" just a nickname with no official status? BTW, I have never heard of a city not coinciding with an administrative unit. Whatever Delhi is, the Wikipedia article should explain it. Wakablogger2 (talk) 21:08, 12 April 2009 (UTC)[reply]
I've linked metropolis in the lede of the Delhi article. Does that help? Who then was a gentleman? (talk) 21:14, 12 April 2009 (UTC)[reply]
Thank you, but that assumes (1) that the claim that Delhi is a metropolis is correct, and (2) still does not provide the actual status of Delhi as the definition of metropolis is vague. If Delhi is a metropolis, is it a large city within the NCT or is it a group of cities in and around the NCT? Some sort of proof is required. Wakablogger2 (talk) 21:19, 12 April 2009 (UTC)[reply]
You yourself said that what Delhi is is variable and uncertain, so what's wrong with defining it as a "metropolis"?
Clearly the usage of the term varies depending on context and usage, and I doubt you can ever define it as "within a 21 km radius from the Presidential Palace" or anything like that. --PalaceGuard008 (Talk) 02:58, 13 April 2009 (UTC)[reply]
I did not say that. I said I'm guessing. The point is that the article pretends like it is giving facts, when it is not. If Delhi is a vague term, the article should make note of that fact. If "approximately a 21-km radius from the Presidential Palace" (to use your example) matches actual usage, then that is the description that should be given. Wakablogger2 (talk) 05:56, 13 April 2009 (UTC)[reply]
If, as you say, you've made a number of posts on the talk page without luck, I would suggest you be bold and make the changes you think the article needs. If someone else has a better idea, they can come along and discuss it, but you will have done your part. --Richardrj talk email 07:43, 13 April 2009 (UTC)[reply]
I have gone ahead as Richardrj suggests and made the following addition/change: The exact status of the name Delhi within India requires clarification, but it appears to be an abbreviation of the "National Capital Territory of Delhi," a federally-administered union territory. Delhi also refers to the general urban area in and around the National Capital Territory (NCT) and is often used to refer to the capital of India, known more properly as New Delhi (also located within the NCT). Wakablogger2 (talk) 18:40, 13 April 2009 (UTC)[reply]
It seems to me that Dehli can mean different things at different times, depending on how it is used and when it is used; maybe it should be thus noted on the article. Your change seems appropriate.96.53.149.117 (talk) 17:49, 16 April 2009 (UTC)[reply]

William Crookshank - Died 1865

I have a document that is a burial certificate for the above person. It originates in Salford Manchester, England. Can anyone tell me more about it and William Crookshank pleaseĀ ? Is he famousĀ ? ā€”Preceding unsigned comment added by AvrilH (talk ā€¢ contribs) 20:20, 12 April 2009 (UTC)[reply]

We have William Cruikshank, but not really Crookshank... AnonMoos (talk) 02:02, 13 April 2009 (UTC)[reply]
By googling ' William Crookshank 1865 Salford ' you can ascertain that he is not famous.--Wetman (talk) 02:19, 13 April 2009 (UTC)[reply]

19th Century German Metaphysics

Lines from George Soros's "The Alchemy of Finance":

"Once you leave the confines of scientific method you are in constant danger of getting lost in a world of your own creation and leaving reality far behind. It has happened before, in medieval theology and in nineteenth-century German metaphysics."

What happened to take nineteenth century German metaphysics off course? What articles could I read to get an idea of what he's talking about? I know he's well versed in philosophy, and I'm sure it's a clever reference.NByz (talk) 21:18, 12 April 2009 (UTC)[reply]

The WP articles on Christian Thomasius and Christian Wolff may be a starting point. Oops, not quite the 19th century... --Cookatoo.ergo.ZooM (talk) 21:38, 12 April 2009 (UTC)[reply]
In which case I suggest Schelling and Hegel. --Cookatoo.ergo.ZooM (talk) 21:43, 12 April 2009 (UTC)[reply]
I think this is a general raspberry being blown at german idealism meltBanana 03:33, 13 April 2009 (UTC)[reply]
It certainly could apply to Hegel and that tradition. It could also be a reference to Karl Marx and his materialist metaphysics which descended from Hegel; I'm not sure how much Soros agrees with Marx. Equally, it might refer to anti-rational philosophers like Friedrich Nietzsche and Arthur Schopenhauer. The whole of 19th century German philosophy was one long succession of strange ideas. --Maltelauridsbrigge (talk) 09:38, 16 April 2009 (UTC)[reply]

Need to write down Hindi script

I've made a transliteration of a Roman letter word into Devanagari script on a few websites and I've managed to get a result, but when I try to copy-paste it into MS Word 03 (it pastes as-is everywhere else!!) it appears as lots of boxes. Now I've tried to do this before and I've been able to use Insert>Symbol with the font Mangal to replicate what the result was manually, but with this word I cannot do that. Um, can someone tell me how I can paste it properly, or what order and number of symbols I need to input to get it? The transliteration is ą¤†ą¤°ą„ą¤šą„‹ą¤ą¤øą„ą¤¤ą„ą¤°ą¤°ą„‡ą¤². It should be coming out as 'Arcoestrela' or similar. Thanks!! Lady BlahDeBlah (talk) 23:29, 12 April 2009 (UTC)[reply]

This is more of a Computing and/or Language problem. However, the characters display with the Arial MS Unicode font, but I don't think they're entirely correct. For example, the first "r" is not transcribed, the next consonant comes out as a "ch", etc. AnonMoos (talk) 02:45, 13 April 2009 (UTC)[reply]
Dang, I didn't see the Language section...poo, should I move the question then? Um, I unfortunately cannot understand the Devanagari script much. I used a couple of the online translators on the external links of Devanagari Transliteration and they both came out with the same result, so I have no idea how right or wrong it is. Lady BlahDeBlah (talk) 13:51, 13 April 2009 (UTC)[reply]
I'd just like to say, I've posted this into the Language section. Lady BlahDeBlah (talk) 22:17, 13 April 2009 (UTC)[reply]
The /r/ is there: it's the rightward hook after the feather of 'o'. (Yes, they really do write it that way.) ā€”Tamfang (talk) 03:43, 14 April 2009 (UTC)[reply]
If Arcoestrela is a Spanish word (or at least pronounced like one), spelling it with k would get a more accurate transliteration. Also I now notice that the second r is transliterated twice, as part of the cluster str and then independently, resulting in ārčoestr(a)rel(a). ā€”Tamfang (talk) 02:29, 15 September 2023 (UTC)[reply]


April 13

New South Wales

Why was New South Wales called New South Wales instead of New Wales?96.53.149.117 (talk) 06:13, 13 April 2009 (UTC)[reply]

Actually, it was called New Wales. New South Wales tells me: In the journal covering his survey of the eastern coast of the Australian continent, Cook first named the east coast of Australia "New Wales", which he later corrected in his journal to "New South Wales".
But why he changed it, that's anyone's guess. -- JackofOz (talk) 09:51, 13 April 2009 (UTC)[reply]
I thought there might have been another "New Wales" already existing somewhere on the Northern Hemisphere, but I didn't find anything conclusive. The article on Welsh American has:
"On a plaque mounted on the east faƧade of the imposing Philadelphia City Hall, the following inscription is found:
Perpetuating the Welsh heritage, and commemorating the vision and virtue of the following Welsh patriots in the founding of the City, Commonwealth, and Nation: William Penn, 1644-1718, proclaimed freedom of religion and planned New Wales later named Pennsylvania. [...]"
The article on William Penn has:
Penn first called the area ā€œNew Walesā€, then "Sylvania" (Latin for "forests or woods'"), which Charles changed to "Pennsylvania" in honor of the elder Penn.
Since the the American New Wales was renamed almost a century before Cook thought about the Australian region, I doubt it had any influence on Cook's decision. Maybe there was yet another New Wales? ---Sluzzelin talk 12:56, 13 April 2009 (UTC)[reply]
Maybe he was distinguishing it from New North Wales? Although there was another New South Wales in the same area already. Adam Bishop (talk) 15:39, 13 April 2009 (UTC)[reply]
We also have Nova Scotia and New Caledonia, which despite being named after the same place, are about as antipodean as you can get. --Jayron32.talk.contribs 00:14, 14 April 2009 (UTC)[reply]
But then, they weren't so named because of any proximity to Scotland, but because of something that reminded the discoverers of Scotland, or because they just wanted to perpetuate Scottish influence abroad. Same deal for the various New Englands, the two New Irelands, New France, and New Britain. I was taught in school that the cliffs of the NSW coast reminded Cook of the cliffs of South Wales, and that was why he thought of the name. But I can't vouch for that. -- JackofOz (talk) 04:33, 14 April 2009 (UTC)[reply]
Obligatory Mitchell and Webb sketch. Seems weirdly similar to what Jack was taught at school. 217.43.141.59 (talk) 16:06, 16 April 2009 (UTC)[reply]

Legend with Hakon and Sigrid?

Hello, I'm reading a novel so I can write about it for Wikipedia but I don't recognize all of the allusions in it. There is one story in the book that is presented as a legend from Norway. There's one time parameter: the novel is set in the 12th century, so presumably the storyteller wouldn't tell the other characters a legend that postdates the period.

The gist of the supposed legend is that there's a woman named Sigrid who is charged by her aged warrior father to brew a potion of invincibility for him, so that he can defeat a certain Hakon Swendson. There is a long-standing feud between the families, but the two meet and fall in love, Ć  la Romeo & Juliet, except with a happy ending.

Does this tale sound like any actual legend from Norse or Scandinavian mythology? A regular Google search turns up lots of Sigrids and Hakons but nothing that fits this story. Thank you. LovesMacs (talk) 06:44, 13 April 2009 (UTC)[reply]

The names reminded me of Hagbard and Signy, but that legend is rather miserable, and Romeo and Julietish in the sense that they both die. So, a pointless answer, really, since it bears no relation to the question. Gwinva (talk) 08:49, 13 April 2009 (UTC)[reply]

Bible as book

Is the bible normally cataloged as a fictional or non-fictional book?--80.58.205.37 (talk) 12:54, 13 April 2009 (UTC)[reply]

Really, it depends on one's religion and one's philisophical stance on the Bible. For example, a creationist would probably take the whole Bible literally, a Christian whole supports evolution would believe the New Testament but take the Old with a grain of salt, and a atheiest would think the whole thing's ridiculous Library Seraph (talk) 13:15, 13 April 2009 (UTC)[reply]

The OP was asking about how the Bible is catalogued, not about philosophical stances. In the Dewey Decimal System, modern translations are given the shelfmark 220.5, meaning it is classed as non-fiction.
Bear in mind that the classification "non-fiction" does not automatically equate to "fact". The writings of David Icke, for example, are also catalogued as non-fiction. Malcolm XIV (talk) 13:19, 13 April 2009 (UTC)[reply]
I would guess most libraries/book shops etc. have religious sections. Some take it upon themself to move them to the fiction section chandler Ā·Ā·Ā· 13:53, 13 April 2009 (UTC)[reply]
I just checked the Swedish library system, where they classify it as "Ccb", which means that they put it specifically in a section called "Religion" (same as The Epic of Gilgamesh and the Qu'ran, I suppose) 83.252.174.221 (talk) 19:02, 13 April 2009 (UTC)[reply]
In BS" section. Is "BS" considered fiction? Edison (talk) 19:15, 13 April 2009 (UTC)[reply]
No, "B" is "Philosophy, psychology, religion", with "BS" simply "The Bible". Fiction vs. non-fiction doesn't really enter into the classification scheme. Most books that would be classified as "fiction" under other schemes wind up in a subcategory of "P" "Language and Literature". - Nunh-huh 22:27, 13 April 2009 (UTC)[reply]
But was some early cataloguer making a wry editorial statement by declaring The Bible to be "BS?" Edison (talk) 03:52, 14 April 2009 (UTC)[reply]
It is tempting to think so, but I suspect it's just one of those amusing coincidences. These days we'd probably skip from BR to BT just to avoid the matter. - Nunh-huh 04:15, 14 April 2009 (UTC)[reply]

What is the general opinion of the Sudan People's Liberation Army in the US and Europe? Are they considered "good guys" or "bad guys," if such a distinction makes sense? Have they been accused of genocide or terrorism?

Thanks, ā€” Sam 63.138.152.238 (talk) 13:32, 13 April 2009 (UTC)[reply]

There's no such thing as a "general opinion" of something like this. Some people will say they are a force for good and others that they are a force for bad. It's impossible ā€“ and anti-intellectual ā€“ to generalize. --Richardrj talk email 14:25, 13 April 2009 (UTC)[reply]
I'm not sure I agree with that. For instance, Afghanistan, 2001, the Taliban were generally considered by the US and Europe to be "bad guys," while the Northern Alliance were generally considered "better guys," or at least "not-so-bad guys." In Sudan, the Janjaweed are generally considered "very bad guys" by the US and Europe -- the US declared that they committed genocide, the US Sec. Council called for them to be disarmed and the International Criminal Court has filed charges against their leaders.
I'm all for moral relativism, but there is frequently a general consensus of opinion on who the West supports.
If the SPLA specifically garners no such consensus of opinion, because their actions are considered ethically "grayer," that would be an adequate answer to my question. ā€” Sam 63.138.152.238 (talk) 15:43, 13 April 2009 (UTC)[reply]
I don't think there is a general opinion. To be honest, in North America at least, no one really cares about Africans killing each other. If anyone even knows that anything bad is happening in Sudan at all, it's probably because of George Clooney. Adam Bishop (talk) 15:35, 13 April 2009 (UTC)[reply]
How about the leaders/diplomats/Human rights organizations of the West? For instance, like I mentioned above, the Janjaweed are generally considered "bad guys" by the West, even if most people don't know who they are. ā€” Sam 63.138.152.238 (talk) 15:47, 13 April 2009 (UTC)[reply]
Also, remember that when the Taliban was the Mujahideen, and was killing the commies, they were the Good Guys according to the West. Likewise, when Saddam Hussein was killing the Iranians who overthrew the Shah of Iran, himself a U.S. ally, he was a Good Guy. Fast forward 20-20 years, and now they are the Bad Guys. For the policy makers and the diplomats in all countries, the "Good Guys" are defined as people who best serve the interest of those policy makers, and the Bad Guys are those that do not; it has little to do with the actual actions of those Guys, only with how they can be use to further the politics of the policy makers. Thus, Taliban in 1980's = Good Guy. Taliban in 2000's = Bad Guy. Saddam Hussein in 1980's = Good Guy. Saddam Hussein in 1990's = Bad Guy. It is this sort of actions by western governments, especially the U.S., that likely leads to the bad reputation the U.S. government has in the world with regards to interventionism. If the U.S. intervened on "moral" grounds, it would be one thing. Its when the U.S. clearly acts out of self-serving interests (and in some cases, takes NO action in places where we have no interest, c.f. Sudan, Rwanda, etc.) that it becomes clear that "Good" and "Bad" on an absolute scale mean nothing for the policy makers. --Jayron32.talk.contribs 00:09, 14 April 2009 (UTC)[reply]
The Taliban are not the same as the Mujahideen... AnonMoos (talk) 11:28, 14 April 2009 (UTC)[reply]
The leadership was different, but the core group was essentially the same. Many of the same people who were American allies as the Mujahideen became American enemies as Taliban. There is a direct evolutionary link between the two groups, but yes, technically the Taliban was a distinct (but descendent) group from the Mujahideen. --Jayron32.talk.contribs 12:42, 14 April 2009 (UTC)[reply]
The Mujahideen became the Northern Alliance, not the Taliban. ā€”Preceding unsigned comment added by 209.251.196.62 (talk) 12:56, 14 April 2009 (UTC)[reply]
Its not a direct "a to b" linkage between either group. The leaders of the Mujahideen largely became the leaders of the Northern Alliance, but in 1980 the U.S. Government drew no distinction between the native Afghan groups fighting the Soviets. Its really a complicated situation; but during the 1980 Soviet war, the Americans supported ALL native groups who opposed the Soviets, including the Mujahideen leadership (later the Northern Alliance) and those that later became the Taliban. --Jayron32.talk.contribs 13:05, 14 April 2009 (UTC)[reply]

Fable About Thinking Ahead

I seem to recall having read a fable/allegory/parable several years ago about prisoners being forced to carry supplies from one Greek city to another. Most prisoners chose relatively loaded sacks of pots/pans to carry while one of them chose the heaviest sack and was scorned for being an idiot. However, it turns out that this sack was full of food, so by the time the trip ended, there was little food to carry and so this prisoner was barely carrying anything at all while his comrades were still carrying the original loads.

Does anyone know what the name of this story is and where I could find it again? ā€”Preceding unsigned comment added by 15.195.201.86 (talk) 17:47, 13 April 2009 (UTC)[reply]

The story is from the life of Aesop and involves slaves (including Aesop himself, the clever fellow with the bread) rather than prisoners. John M Baker (talk) 21:45, 13 April 2009 (UTC)[reply]

native american population history

where is our article on the history of the population of native amerincas, please? I glanced over Native_Americans_in_the_United_States and also read the article titles in the "see also" section at the end but can't seem to find the article. (I am especially interested in the reduction over time in their numbers, if this happened, and the referenced causes). Thank you. 94.27.151.13 (talk) 20:32, 13 April 2009 (UTC)[reply]

You may want to have a look at the article on population history of American indigenous peoples. --62.47.129.162 (talk) 21:02, 13 April 2009 (UTC). Oops, --Cookatoo.ergo.ZooM (talk) 21:09, 13 April 2009 (UTC)[reply]
The OP may want to head to a bookstore or library and check out 1491: New Revelations of the Americas Before Columbus, a very accessible and enlightening read on this topic. Other books on Native American population history which are good are Guns, Germs, and Steel and The Columbian Exchange. Additionally this short article has some great references you may want to follow. --Jayron32.talk.contribs 12:59, 14 April 2009 (UTC)[reply]
Note that "1491: New Revelations..." is not an uncontroversial book but our article lacks a critical reception section or any indication at all of any review of the work. (The "Guns, Germs and Steel" article has some such information.) Rmhermen (talk) 20:51, 14 April 2009 (UTC)[reply]

Placing blame elsewhere

What is the name for the observed human tendency to fit newly observed data into a person's current set of assumptions, no matter how much the data conflicts with the assumptions?

  • A driver runs a stop sign and slams into a pedestrian, and instantly places blame on the pedestrian for getting in the way (fitting the belief, "I am a good driver");
  • a Republican in America blames the current recession solely on the policies of President Clinton although there were 8 intervening years of Republican governance (fitting the belief, "Republicans are better rulers than Democrats");
  • a Democrat in America blames the Republicans for warrantless wiretapping although it has been approved repeatedly by Democratic lawmakers (fitting the belief, "Democrats are better rulers than Republicans")

The political examples above are intended as examples and not as flamebait, hence my use of both sides. Thanks - Tempshill (talk) 23:16, 13 April 2009 (UTC)[reply]

Confirmation bias may be what you mean. However, I feel I should point out when it comes to politics, that may not really be the case a lot of the time. I suspect it doesn't have a whole lot to do with psychology -- at least in terms of people displaying an unconscious tendency. They have an agenda, and they're very much aware of how important it is to manipulate the media and voters. That's not the same thing as actually believing in what they say. This is not to say that confirmation bias and other similar psychological effects aren't a factor in politics, but a lot of politics is about putting a spin on things regardless of what you believe. -- Captain Disdain (talk) 23:40, 13 April 2009 (UTC)[reply]
That's what I'm looking for ā€” and it looks like a good article, thank you. Tempshill (talk) 23:54, 13 April 2009 (UTC)[reply]

Also cognitive dissonance... AnonMoos (talk) 04:27, 14 April 2009 (UTC)[reply]

Buck passing. --88.110.37.123 (talk) 08:14, 14 April 2009 (UTC)[reply]

April 14

Kufuor family

This website is about John Kufuor's family. They said John Kufuor have 9 siblings, 4 males 5 females, Nana Agnes Addo Kufuor is the oldest sister. So who is the oldest siblings in John Kufuor's family, or John A. Kufuor is the oldest child. Is Kwame Kufuor the second child in Kufuor's family? Then who is the youngest? and what age is the youngest. One of his kids go to school in Louisiana, does he have any family living in California?--69.226.37.237 (talk) 00:37, 14 April 2009 (UTC)[reply]

a more direct link to the page in question ā€”Tamfang (talk) 03:33, 14 April 2009 (UTC)[reply]
Isn't these two links the same. I'm looking for a link is more specific about Kufuor's family ancestory. Who is the oldest child in Kufuor's family, and over his kids what's the earliest birth range. Is his oldest child born in 1965, youngest in 1973? This site don't say much. Does Kufuor have any family ancestorys in California?--69.226.37.237 (talk) 03:43, 14 April 2009 (UTC)[reply]
The two links will show you the same page, but one of them unnecessarily goes to Google first. It's not important but I like to simplify such things whenever I can. ā€”Tamfang (talk) 05:23, 15 April 2009 (UTC)[reply]

Partial answer: Brittanica says "John Kofi Agyekum Kufuor was the 7th of 10 children and the son of Nana Kwadwo Agyekum, an Asante royal, and Nana Ama Dapaah, a queen mother". The BBC confirms (or simply repeats) this statement. I haven't found the names of his elder siblings in another source. Hopefully another editor can answer better, including the part about Kufuor's children. If you have library access, you might look for one of the few published biographies of Kufuor. Best, WikiJedits (talk) 15:38, 14 April 2009 (UTC)[reply]

    • Does 7th of 10 kids means 6 siblings is older than J.A. Kufuor? The links I posted above named only 7 out of 9 siblings.

  • Nana Agnes Addo Kufuor - Eldest Sister, spouse of the late Asantehene,
  • Sir Nana Osei Agyeman-Prempeh -Professor Emeritus Francis Addo-Kufuor -2nd born; Chairman of the University Council of Kwame Nkrumah University of Science and Technology
  • Hon Dr Kwame Addo Kufuor -Defence Minister
  • Mrs Rebecca Dodoo - wife of the late Registrar of the University of Ghana. Lives in the United Kingdom
  • Mrs Cecilia Campbell -lawyer in the United Kingdom
  • George Addo Kufuor -Businessman
  • Mrs Josephine Kankam -Kumasi

Maybe 2 of his sibling is dead. The others have no ages is listed. maybe this is normal women habit to keep personal on agings.--69.228.147.37 (talk) 23:13, 14 April 2009 (UTC)[reply]

Yes, 7th of 10 would normally mean six were older and three younger. I've since found this book, as well. Not all of it is available, but the part I could read was about Kufuor's mother, and lists nine (not ten) of her children, (though it doesn't specify their birth order) as Francis Addo, Agnes Addo, John Agyekum, Kwame Addo, George Addo, Mariam Addo, Cecelia Akoto, Rebecca Doudu and Josephine Annin. It also mentions an alternate name for John - Kofi Diawno/Diawuo which you might also try googling.
Also found this newspaper article from 2002 (scroll down to "President Kufour: still gentle, still open"). In it he says he had four brothers (Professor Emeritus Francis Addo-Kufour, George Addo Kufour, other two not named) and five sisters (Nana Agnes Addo Kufour, Mrs Rebecca Dodoo, Mrs Cecilia Campbell, Mrs Josephine Kankam, other one not named) and that one sister was by that time dead. (NB that this confirms the ten siblings number, meaning the book left out one brother.).
The newspaper article also says his eldest child is his son J. Addo Kufour who was born about 1964. The younger four children are named also. You may have to continue googling each person's name individually to try and find more. Best luck, WikiJedits (talk) 11:26, 15 April 2009 (UTC)[reply]
  • Thanks, we have all informations about John's siblings. John Kufuor's siblings also have their own kids (The only well known is John K. Agyekeum and Kwame). Total John Kufuor have 10 grandkids. Going on google and just dig for such a gold/diamond like this is tough. Where they live? Everybody is all over the earth. He have family in England, one in Louisiana, I'm wondering if any of his families is in California? Los Angeles? Orange County?--69.231.5.91 (talk) 22:30, 15 April 2009 (UTC)[reply]

magazine archives

I'm currently trying to find a copy of an article Willie Morris wrote for Reader's Digest. The article was about his black Labrador Pete. I tried searching the Reader's Digest website. But I found nothing. If anyone out there can help me, I'd appreciate it very much. Thank you.69.203.157.50 (talk) 05:09, 14 April 2009 (UTC)[reply]

You're looking for Willie Morris, "Pete, the mayor of Bridgehampton," Reader's Digest December 1978, according to William Thomas, "Willie Morris, Home Again" reprinted in Conversations with Willie Morris. Morris's "Preface" to The Courting of Marcus Dupree, 1992 begins "It has been ten years since my beloved Black Labrador Pete,long since departed to the Lord's infinite Heaven for the finest and bravest ..." But you knew that... --Wetman (talk) 07:06, 14 April 2009 (UTC)[reply]
I work for a magazine, and if anyone contacted us with such a specific reference - issue, year, and name of article - we could easily find it in the archive and would be pleased to mail out a photocopy. I don't know if Reader's Digest does this (perhaps their reader letter volume is greater than ours) but it could be worth a try now Wetman has provided the details. WikiJedits (talk) 15:41, 14 April 2009 (UTC)[reply]
The reference librarian at even a medium sized public or college library could tell you if they have it in their collection, or what nearby larger libraries have it. I have seen decades of Reader's Digests on the bookshelves of a small college library. You might also be able to buy it on Ebay, since right now three copies of the December 1978 issue are offered for sale right now:_Unfortunately I cannot link directly to the page with the three listings because of our annoying "spam filter," but just go to www.ebay.com and enter Reader's Digest December 1978 in the "Find" window.. Edison (talk) 21:24, 14 April 2009 (UTC)[reply]
I'd expect that any good-sized US library would stock old Reader's Digests (although some of the older ones might be well digested by now). Once you find the article, you could make copies. StuRat (talk) 21:36, 14 April 2009 (UTC)[reply]

under the gold standard, is it much easier to launder money?

under the gold standard, is it much easier to launder money, like a simple soap-and-water affair?


or is it that even if you were to go so far as melting it isotopes or something else would still give you away?
(obviously for both questions above, i mean you or whoever you were getting money from would turn in the notes for the gold, either for you to sell on to someone else to turn into notes again, or you can do it yourself, if i understand how the gold standard works...) ā€”Preceding unsigned comment added by 79.122.35.239 (talk) 05:28, 14 April 2009 (UTC)[reply]

Anti-money laundering relies not so much on tracing the physical notes and coins, but the flow of the value of the money in an accounting sense. Buying metal and then selling the metal to convert from "dirty" to "clean" cash is generally just as (in)effective whether under the gold standard or not. Generally speaking, you can as easily go out and buy a tonne of gold as you could back in the days of the gold standard. The only possible advantage under the gold standard is that you are more likely to end up with the same amount of (say) US Dollars you started with under the gold standard (since one is pegged to the other), while today you would face the risk of the price of gold changing. In a market where gold prices are going up, though, this is probably a good thing. And if you are willing to spend a little, you can insure against the risk of the price movement. --PalaceGuard008 (Talk) 07:55, 14 April 2009 (UTC)[reply]

Money laundering is hiding the origin of money so as it is not possible for authorities to show that the millions you have in the bank is not the result of a bank robbery but rather the result of a prolonged series of wins in Vegas. Essentially the individuals look to take the money they have received by dubious means and then get to a point where they can say "this is money taken from x" - so for instance a common technique is for a large amount of money to be placed into an Investment, then the investment is surrendered at which point you get your money back - less any surrender charges. This gives your money a 'history' which can explain where the money you have comes from (of course it can take more than one 'cycle' to get the money 'clean' but you get the idea). What you mention is a technique to launder money too but it isn't reliant on the gold standard. Most money laundering will result in a reduction in return - it's the 'price' the person pays to try clean the money and short-term notable-losses are something that financial institutions look out for when trying to uncovered money laundering. 194.221.133.226 (talk) 08:26, 14 April 2009 (UTC)[reply]

Puritan Pilgrims

The Puritans were not allowed to worship freely in England, so they left to form colonies in the New World. But since the 13 colonies were under the same British rule as the England they had left, why were they allowed to worship freely in the colonies? ā€”Preceding unsigned comment added by 23SarahBeth (talk ā€¢ contribs) 06:18, 14 April 2009 (UTC)[reply]

A fundamental difference between the policies of Britain vs. France and Spain in the 17th century etc. is that the French and Spanish governments imposed strict religious orthodoxy on their colonies, while the British authorities were generally glad to see dissidents and malcontents sailing off to English colonies (an easy way to get rid of them from the mother country, while building up strategic English settlements abroad)... AnonMoos (talk) 10:56, 14 April 2009 (UTC)[reply]
The OP may want to read our article on Plymouth Colony and Massachusetts Bay Colony and Pilgrims and Puritans for some background on religious tolerance and Puritans in British North America. As a bit of a background, King James I called the Hampton Court Conference to deal with the "Puritan question". After this conference, things became "unpleasant" for Puritans in England as they were regularly rounded up and jailed for their religious beliefs. Some of the Puritans moved to the Netherlands (henceforth known as Pilgrims), since Religious tolerance was much greater there, and the Dutch had a Calvinist belief system that more closely matched their own. The Netherlands was not an ideal solution, however. In the first, the Pilgrims that moved there were not Dutch, and so never quite "fit in" to Dutch society. Secondly, they weren't exactly free from persecution of the English. Since the Pilgrim leaders in the Netherlands kept agitating for religious change in England, the English authorities would show up in the Netherlands to arrest them from time to time. So, the "colonial solution" seemed a good deal for all parties. The English could essentially export the agitators to another continent, where they were isolated from the motherland, and could cause no more problems. The Pilgrims could also worship in peace.
The model worked so well in Plymouth that it was done over and over. Massachuestts Bay Colony was founded by Puritan reformers (a different group from the Seperatists that formed the core of the Pilgrim movement). The Colony of Rhode Island was founded by dissenters who had been kicked out of Massachusetts Bay and Plymouth. The Colony of Pennsylvania was founded as a home for Quakers and the Colony of Maryland was founded as a Roman Catholic haven. Of course, there were other reasons for founding colonies. The Colony of Virginia had been founded as a corporate commercial venture, The Province of New Hampshire was a private land grant to Captain John Mason and the Province of Georgia had been founded as a sort of giant debtors prison colony. --Jayron32.talk.contribs 12:35, 14 April 2009 (UTC)[reply]
The pilgrims and puritans and other religious groups did not come to America because they believed in freedom of religion for any but themselves. They persecuted dissenters from their views if anyone dared to openly worship other than in the official approved manner of the colony. The persecuted all too quickly become the persecutors. Edison (talk) 21:09, 14 April 2009 (UTC)[reply]

voting machines

After reading the article on Diebold and Premier Election Solutions, I have to know, are these machines still going to be used for elections? Is anything being done to fix this? If there is, why isn't it mentioned in the article? thank you ---Jeevies (talk) 09:09, 14 April 2009 (UTC)[reply]

Sorry, it seems that I've been mistaken in my wording. I"m not asking about the debate of using machines against paper ballots at all. I'm asking about the particular Diebold/Premier kind of voting machines, as I was shocked after reading all the controversy listed with them. So what I want to know is if anything has been done to address these particular brand of voting machines since the article does not list any. The other question I asked is that after all these points of controversy, is this company still at large? --Jeevies (talk) 20:51, 14 April 2009 (UTC)[reply]

How about answering the question instead of debating the merits of paper vs electronic vs phone vs mail balloting? Jeevies, I dont have time right know to track down all the infos you need but off the top of my head- yes they are still in business. I remember reading an article (NY Times iirc) discussing a software patch that was to have remedied much of the problem. Hopefully someone will come along that can get you some links. 161.222.160.8 (talk) 21:42, 14 April 2009 (UTC)[reply]
Gee, Mr IP Editor 161.222.160.8, people editing from your anonymous address have had five vandalism warnings. How credible does that make you as to the best way to answer Ref Desk questions? Edison (talk) 05:08, 15 April 2009 (UTC)[reply]
Gee, Mr Registered editor Edison, been to the beach lately? It was just this guy was asking a question, coming back to restate his question, and all he was getting was personal opinion on the accuracy of varius voting methods. Was just pointing that out. And yeah, AGF sucka 161.222.160.8 (talk) 23:49, 15 April 2009 (UTC)[reply]
The problem appears to be mostpeople are answering/concentrating on Malcolm XIVs supplemental question so I've moved that into itsown subsection Nil Einne (talk) 16:51, 16 April 2009 (UTC)[reply]
Please assume good faith. Having said that, if you're willing to read a lot of Government Speak, this page goes into the California Secretary of State's approvals/disapprovals of elections systems. Who then was a gentleman? (talk) 02:07, 15 April 2009 (UTC)[reply]

Paper balloting vs electronic

I moved this into its own section because of concerns by the OP that supplemental question was detraction from his/her question [11] Nil Einne (talk) 16:51, 16 April 2009 (UTC)[reply]

As an additional question, what exactly is the problem with putting an X on a piece of paper? Why are these machines needed at all? Malcolm XIV (talk) 09:48, 14 April 2009 (UTC)[reply]
Well if they worked correctly a machine would be more 'secure' than a piece of paper. A machine would allow for faster return of results(and remove risk of miscounts). A machine would (potentially) require less labour than paper-ballot and a machine could even maybe be sold as a 'green' idea if they are used many-many times. A machine can also be kept accurate more quickly. A machine could also introduce more individual-verification (you could be mailed a unique code to identify you that then must be input to vote). If properly implemented such an item could offer many advantages to paper-ballots. 194.221.133.226 (talk) 11:08, 14 April 2009 (UTC)[reply]
We had a discussion on these contraptions around the time of the US presidential election, see here. --Richardrj talk email 09:52, 14 April 2009 (UTC)[reply]
Am I right in presuming you're not an American? If so an important point to realise which I think many non Americans don't (at least I didn't use to) is how many things Americans actually vote on in their elections. It's not just for the President, Senator, Congressperson, state Senator etc but potentially stuff like school board members, judges, coroners, sheriffs, potentially multiple amendments to the state constitution and other referenda, and other stuff I forget. Take a look for a ballot and you may start to get an idea. Also Americans are still used to having definite results on the night Nil Einne (talk) 10:44, 14 April 2009 (UTC)[reply]
Like in Florida? ā€”Preceding unsigned comment added by 209.251.196.62 (talk) 12:10, 14 April 2009 (UTC)[reply]
Another factor is the tabulation of the votes, both in terms of accuracy and in terms of logistics. I did vote a paper ballot once (the mark-with-an-X kind, not the optical-scan kind), but that was in a county of 5,000 people. There are about that many people who live in my Maryland voting precinct (granted, not all of them voters), and 300 such precincts in my county. In the 2008 general election, there were 427,500 votes cast in the county. Just in my precinct, you had these choices: president (6 candidates), state attorney general (3), state auditor (3), state treasurer (3), state judgeships (4), U.S. congressional representative (4), state senator (2), state representative (8), county board of education (2), proposed amendments to the county charter (2), county referendum (1), and two proposed amendments to the Maryland constitution. Nine months earlier, we had the primary. That's a lot of paper. --- OtherDave (talk) 16:41, 14 April 2009 (UTC)[reply]
The Florida case is actually an excellent example. Beyond all the other controvery, I think it was clear that the idea that they didn't know for sure who their next President was one day later let alone one month later was deeply unsettling for many Americans. And one of the reasons for the controversy was the many deadlines. As far as I'm aware, in many countries deadlines of that sort are non existant, the most important thing is getting an accurate result and if problems arise they need to be sorted in some way that is fair, no matter how long it take. Timeliness is important but it always takes second fiddle to accuracy. Of course Americans do have some definite deadlines, e.g. when the President is supposed to be sworn in whereas in Westminister style parliamentary democracies, there is no definite deadline and it is accepted that the caretaker government lasts until someone is able to form a new government (which can sometimes take months). Nil Einne (talk) 16:43, 16 April 2009 (UTC)[reply]
The key thing is to have a voter-verified paper trail. I have no problem with an electronic voting machine which marks a paper ballot on a roll of paper inside the machine which I can read before I press the key agreeing to "cast" the ballot. The objectionable type of machine just keeps an electronic total with no voter-verified paper trail, and it has been demonstrated to be quite possible to alter the totals. If a voter marks a paper ballot, what do we do with the ones where the voter marks it illegibly or ambiguously? In past elections, it was also possible for a crooked election judge to spoil a ballot marked for the "wrong" candidate by adding another mark by a different name with a piece of pencil lead concealed by a bandage around his finger. In some elections, there are literally dozens and dozens of referenda, water and sewer board members running unopposed, and judicial retention ballots running to many pages, and it would be a nightmare to accurately and repeatably tabulate them. In Florida in 2000, we saw that voters were not even competent to push a button hard enough to punch a punch card. I recommend secret ballots, with no internet or phone or mail voting without a good reason, and machine marked voter-verified ballots. Whenever the ballot is marked or the vote is cast from home, there is a liklihood of abuse by having a political hack "help" the voter while he casts the ballot from home. Absentee ballots should be limited to those unavoidably away on election day, since we have no way of knowing who is watching over the voter's shoulder. I recommend that the initial and official count be from the machine marked roll of paper which the voter saw and approved, rather than saving that actual ballot for possible recounts, to avoid hanky-panky in the computer on election night. Edison (talk) 21:02, 14 April 2009 (UTC)[reply]

Normandy

Who might have succeeded William II as Duke of Normandy, had he been killed at hastings? My understanding of the situation is that he had a considerable struggle to gain the Dukedom for himself, and would then have left a young son as his apparent successor. Was there anyone else that would likely have attempted to seize power at the time? 88.108.2.248 (talk) 13:58, 14 April 2009 (UTC)[reply]

Historian Harriet Harvey-Wood argues that the death of Harold Godwinson on the field at Hastings was the decisive factor in the Anglo-Saxon defeat. Consequently, if William of Normandy had died at the point in the battle where he was (first?) unhorsed and mistakenly thought dead, the Normans would have lost. If Harold had lived, he would have remained king; if Harold had also died, the most likely candidate for king would have been Edgar Atheling, who although young was the witan's chosen successor to Harold.
Robert Curthose was aged either 15 or 12 in 1066, and was William's legitimate son, so he might well have held the loyalty of William's Norman subjects. However, Curthose's track record as a leader and warrior is not good; he might easily have relinquished control of the duchy of Normandy to any of his neighbours (Brittany, Vexin, Flanders, France), or to either of his brothers (William Rufus, Henry I). AlexTiefling (talk) 14:54, 14 April 2009 (UTC)[reply]
I agree that Robert Curthose almost certainly would have inherited Normandy, but assuming that William's half-brothers Robert, Count of Mortain and Odo of Bayeux survived Hastings, one (or both) of them probably would have taken control if Curthose was underaged (or, even if he was already of age, since they seem like the kind of people who would have done that). There was a problem with the succession of William himself, since he was illegitimate and only seven years old when his father died, so I would expect plenty of assassinations and meddling from neighbours, as Alex said. Adam Bishop (talk) 15:08, 14 April 2009 (UTC)[reply]
I had forgotten about Odo and Robert - that's a good suggestion. I can envisage a scenario in which Odo takes control and persuades Gregory VII (thought to have been the main source of church support for the invasion) to make Normandy a prince-bishopric. AlexTiefling (talk) 15:14, 14 April 2009 (UTC)[reply]
This History of Normandy (start on page 266) elaborates on the many insurrections and revolts William dealt with in Normandy throughout the 1040s, 50s and 60s, which may yield more suspects. Besides Odo and Robert as mentioned above, there are also William FitzOsbern, 1st Earl of Hereford and Hugh de Grandmesnil, both of whom that book says (page 409) were granted administrative powers by William while he was in England. Anther relative to consider might be Guy I, Count of Ponthieu, son of William's sister Adelaide of Normandy. Best, WikiJedits (talk) 15:24, 14 April 2009 (UTC)[reply]

(out) Wow, this question is quite academic, but I'll try to answer it as best I can.Ā :-) There were plenty of people who wanted power at that time, but it's all guesses as to who would have jumped at it.
If William had been killed at Hastings and his army lost, but Harold Godwinson had been killed also, you would have had a very weak England [missing a king and a good chunk of their army]. Presumably the young Edgar the Ɔtheling would have been proclaimed king, but I wouldn't doubt that anyone wanting freedom from England would pass up the opportunity to split without fear of a major invasion (who would they have invaded with?).
If William had been killed but his army won, a very interesting Ten Thousand (Greek)-type situation would have arose. What the army would have done is questionable. Would they have picked a commander and, if they had enough people remaining, have gone on a vengeful rampage throughout England, sacking cities as they went? Would they have just gone into a total retreat back to their fortress at Hastings and gone home? Would they have deserted the army and settled in the area to be assimilated into English culture? Does anyone have a reasonable guess?
In Normandy, I don't think that Normandy as a separate vassal state would have lasted long. With a good portion of the army in England or laying dead at Hastings, any duke, or maybe even King Philip I of France himself (his father did attempt to invade Normandy in 1054 and 1057), could have invaded the weakened state. William's young son may have indeed been his successor, but I doubt that the little guy would have reigned long.
I hope this helped, but keep in mind that this was all conjecture and subject to interpretation... ā€”Ed 17 (Talk / Contribs) 15:34, 14 April 2009 (UTC) [reply]

I'm not so sure about the separatism issue; Edgar Atheling was young, but he was also popular. The main separatist leader had been Tostig Godwinson, who had died two weeks previously at Stamford Bridge, and there's not much evidence to suggest that any significant group governed by the English crown wanted rid of it at that point. AlexTiefling (talk) 15:59, 14 April 2009 (UTC)[reply]
Maybe not many were there before, but a sudden weakening of England might tip even slightly unhappy barons over the edge. ā€”Ed 17 (Talk / Contribs) 16:18, 14 April 2009 (UTC)[reply]
If both William and Harold had died, would the Norse have attempted another invasion? I'm not sure what they were up to after Stamford Bridge. Adam Bishop (talk) 20:17, 14 April 2009 (UTC)[reply]
Stamford Bridge was a massacre; it devastated the Norwegian army and left both Harald Hardrada and Tostig, the main architects of the invasion, dead. It's often regarded with hindsight as the end of the viking age. Norway never again attacked England, and Scotland gradually absorbed Norway's British colonies. I think that by Hastings, the Norwegians are out of the running. AlexTiefling (talk) 23:33, 14 April 2009 (UTC)[reply]
If both William and Harold were dead and with England massively weakened, would Scotland have attempted an invasion - at least trying to take the north? Warofdreams talk 01:07, 15 April 2009 (UTC)[reply]
Interesting question. Malcolm III does seem to have tried to exert control over parts of northern England during the reign of Edward the Confessor, and to have stayed out of the wars of 1066. It's possible he might have seen this as a crucial opportunity. However, see my further response about Northumbria below. AlexTiefling (talk) 10:51, 15 April 2009 (UTC)[reply]
With the death of Tostig, was there any support for his 14-year-old son Skuli? Skuli's mother was related to the Count of Flanders, and to William by marriage. Who then was a gentleman? (talk) 02:11, 15 April 2009 (UTC)[reply]
I don't think there was any local support for Tostig and his family. Tostig had been outlawed by the Northumbrians, and replaced as earl by their own choice, Morcar. If the Saxon kingdom had not fallen, the overwhelmingly likely outcome in my opinion is that Morcar would have been endorsed as earl, and been able to resist any incursions either by the Scots, or by the Norwegians in the name of either Tostig's or Hardrada's families. AlexTiefling (talk) 10:51, 15 April 2009 (UTC)[reply]

UK or European equivalent to paleoconservatism?

Is there a political movement in the traditionally more liberal European/UK regions that is roughly equivalent to the American paleoconservatism? If so, I'd be interested in reading more about it.--Over2u (talk) 15:10, 14 April 2009 (UTC)[reply]

In the UK, terms like Old Tory carry some similar connotations. I'm not sure about the rest of Europe - what would you say were the distinctive characteristics of palaeoconservatism? AlexTiefling (talk) 15:16, 14 April 2009 (UTC)[reply]
The article explains it.--Over2u (talk) 16:32, 14 April 2009 (UTC)[reply]
Thought I don't know of any close equivalents there are certainly movements that share many values with Paleoconservatives. Most of them are classified as Far Right, sharing a belief in 'traditional values', restriction on immigration, anti-federalism (anti EU), cultural superiority, anti-authoritarianism. Not all are overtly racist. Most don't have a religious focus. DJ Clayworth (talk) 17:01, 14 April 2009 (UTC)[reply]
It seems I don't always know what I'm talking about. This article might be interesting. The British National Party seems to share a lot of paleocon values. [12]. DJ Clayworth (talk) 17:16, 14 April 2009 (UTC)[reply]


I don't think there is any UK equivalent because much of paleoconservatism seems to be unique to the United States, such as what the American republic truly stands for regarding imperialism and federalism. For example, the British Conservative Party have always been the imperialist party. However they do share a belief in social conservatism. No political movement in the UK seems to be paleoconservative.--Johnbull (talk) 17:16, 14 April 2009 (UTC)[reply]

I'd put a case for the UK Independence Party sharing a lot of paleoconservative values - although some members do take more libertarian views on some issues. Warofdreams talk 00:56, 15 April 2009 (UTC)[reply]
As above I'd strongly suggest that the British National Party is equivalent to paleoconservative. They are anti-immigration, anti-federalist (European federalism in this case), pro white culture and according to the above article self-styled "defenders of Christianity". The fact that the BNP is seen as "extreme right" is more of a comment on the difference between US and UK politics than between the two movements. UK Independence is probably a good match too - libertarianism is also a paleoconservative value. But the UKIP is not as strongly pro-capitalist, pro-white-culture as paleoconservatives. I'm not even sure if the UKIP is in favour of immigration restrictions or if it's as pro-military as paleocons. DJ Clayworth (talk) 13:48, 15 April 2009 (UTC)[reply]
UKIP definitely stand for those things - see their policies in brief: "We will freeze immigration for five years, speed up deportation of up to a million illegal immigrants by tripling the numbers engaged in deportations, and have ā€˜no home no visaā€™ work permits to ease the housing crisis" and "We will support our armed forces with more spending on equipment, military homes and medical care. We will save our threatened warships and add 25,000 more troops". Warofdreams talk 23:15, 15 April 2009 (UTC)[reply]
On the BNP - the examples given in the paleoconservativism article are of views which would be held by some in Conservative Party, and by some in UKIP; they'd be considered hard right in the UK, but not generally as beyond the pale. In contrast, the BNP comes from a neo-Nazi background. That, of course, is played down by the BNP, and their actual policy statements may be closer to the paleocons, but our article gives many examples of incidents and statements by prominent members which I (and I suspect many others) would see as the continuing influence of that background. Warofdreams talk 23:31, 15 April 2009 (UTC)[reply]
I'd say that religion is a very strong element in Palaeoconservatism, as indeed it is in American politics as a whole when compared to Britain. And it really isn't a major factor in the policies of either UKIP or the BNP. The latter's stance as self-styled "defenders of Christian values" seems to be far more about being anti-Islamic and anti the cultural relativism of PC than it is about being a bunch of bible-bashers. It's an example of dog-whistle politics. Malcolm XIV (talk) 23:22, 15 April 2009 (UTC)[reply]
Excellent point that I was planning to make myself. Of course I presume in Britain similar to here in NZ, there are parties with a strong 'Christian' foundation and focus, but they tend to lack many of the other features of paleoconservatisim. Also most countries lack any anti-federalism for obvious reasons. Okay it may exist in the UK to some extent but it's clear of a rather different nature (if anything often opposition to fedaralism means they support a strong central government with limited or no independent assemblies for England, Wales, Scotland and [North] Ireland). And ironically the BNP itself of-quoted as an example above appears to support a federation with Ireland. Of course, there is some similarity in opposition to the European Union by some parties. And some parties do strongly support local government/governance but this is common in the left as well as the right. Also as has already been hinted at, many of the conservative movements in the UK (and probably other monarchies) are strongly supportive of the monarchy, something which I think would clearly be antithetic to paleoconservatives Nil Einne (talk) 16:14, 16 April 2009 (UTC)[reply]
There are a number of parties in Eastern Europe such as the Polish People's Party which fit into the socially conservative, agrarian, anti-communist mold of paleoconservatism. Christian democracy in Europe (e.g. the old DC in Italy) has some factors in common but tends to be more moderate: while being anti-communist, right wing, pro-business, and somewhat socially conservative, they tend to have some support for a modest welfare state and other social provisions (similar to One Nation Conservatism in the UK). Paleoconservatives in Europe would tend to be split between mainstream right-wing (Christian Democratic) parties and the nationalist, protectionist, populist, agrarian, extreme right wing groups like the British National Party, French Front National, and the now-defunct Alleanza Nazionale. --Maltelauridsbrigge (talk) 09:54, 16 April 2009 (UTC)[reply]

presidential power and executive orders

If the president of the US wished, could he issue an executive order confiscating American goods that were legally purchased? As an example could the president order the confiscation of all the Dr Pepper in the US, even though that at the time of purchase, it was perfectly legal to buy the Dr Pepper? Would compensation be required, and if so, who would determine how much? 65.121.141.34 (talk) 15:50, 14 April 2009 (UTC)[reply]

In your example, I'm almost sure that that would require an Act of Congress. As far as I know, executive orders only affect people under the president (the executive branch). ā€”Ed 17 (Talk / Contribs) 15:59, 14 April 2009 (UTC)[reply]
Nobody really knows the limits of US presidential power. They are defined by the interpretation of the Constitution that the three branches of government choose to make at a particular time. If the president felt that Dr. Pepper was a threat to the nation, and Congress and the courts declined to challenge him/her, then he would certainly have a free hand to root out the syrupy threat with no compensation required. --Sean 19:06, 14 April 2009 (UTC)[reply]
The Fifth Amendment to the United States Constitution says that he can't take your Dr. Pepper without compensation. Also, the Fourteenth Amendment to the United States Constitution says that the government can't take your property without "due process of law". Upon this massive seizure, I believe any court would agree with a Dr. Pepper-drinking plaintiff that the seizure was illegal under the 14th; that it didn't serve any legitimate state interest and hence did not give due process to us all; the court would issue an injunction ordering the government to return all of the Dr. Pepper. An exception I can think of: If the FDA ruled that Dr. Pepper was an imminent threat to all human health, then the President might be able to order a mass seizure of all the Dr. Pepper in the US, and upon all the caffeinated consumers filing their class action lawsuits, the courts might decide that this not only was justified under the 14th amendment as complying with due process, but also didn't comprise a "taking" under the 5th amendment, so no compensation would be due to you. Here's hoping the FDA stays off our backs. Tempshill (talk) 20:06, 14 April 2009 (UTC)[reply]
You think that the Constitution would stop a president of the U.S. from seizing soft drinks if he believed it was necessary to protect the nation? Franklin Roosevelt's 1942 Executive Order 9066 declared that people of Japanese ancestry could be hauled away at gunpoint by the military and put in concentration camps, even if they only had 1/16 Japanese ancestry. Unlike the internment of enemy aliens from Germany or Italy, this Executive Order applied to U.S. citizens "with one drop of Japanese blood." The U.S. Supreme Court in Korematsu v. United States refused to interfere with this executive order. Perhaps the U.S. government would apologize to the former owners of the confiscated Dr. Pepper 40 years later, and pay reparations, as they did to the interned Japanese-Americans.As for the necessity of a prior acto of congress to empower a president, recent presidents have used acts or resolutions passed in time of crisis as a blanket authorization to take actions not ever explicitly discussed in the legislative history of the empowering "use of force" measure. Edison (talk) 20:44, 14 April 2009 (UTC)[reply]
And Abraham Lincoln suspended habeas corpus and other civil liberties during the War. --Gadget850 (talk) 21:04, 14 April 2009 (UTC)[reply]
(edit conflict) No, I don't dispute that a US president could and would seize all national Dr. Pepper assets if it were suspected of brainwashing America's youth into sympathy for Communism. As with my FDA example, this would be an exception. Under normal circumstances, any court would reverse the seizure as a violation of the 14th. Tempshill (talk) 22:28, 14 April 2009 (UTC)[reply]

I was reading an annotation dealing with presidential powers under Article I this past weekend. Many scholars believe the president has broad scope under the necessary and proper clause. The widest power exists in foreign affairs and the military. Ex parte Milligan said Lincoln could not suspend habeus corpus while civil courts were open. The Supreme Court has refused to hear these cases often, invoking the political question doctrine. Overall, I was impressed by how the actual powers of the president were undefined in many respects. It is only with the advent of the regulated state that courts have begun to hear these cases. The Federalist Paper authors, Hamilton and Madison, took different theories as real political disputes were presented. I have no doubt that in the field of foreign affairs and military Dr. Pepper could be confiscated. Korematsu v. United States is the gold standard, in my opinion. So much depends on the political climate within the United States when the power is asserted. On September 11, 2002, when we were attacked, the president had much broader powers than today. The annotation said powers, not express, may be available to the president to fulfill the office of the executive. Absent a clear-cut case with detailed facts, one never knows what is constitutional. 75Janice (talk) 22:26, 14 April 2009 (UTC) 75Janice[reply]

Abraham Lincoln's disregarding of the Supreme Court during the first three years or so of his term has to do with the very special circumstances that the U.S. was facing its greatest-ever internal crisis at the time, together with the fact that Chief Justice Roger B. Taney was widely reviled in northern politics after the Dred Scott decision of 1857, and regarded as little better than a traitor by many in the north during the last three to four years of his life. There was very little political cost to Lincoln in defying the discredited Taney court, since almost all Taney supporters would be irreconcilable political opponents to Lincoln, regardless of whether Lincoln paid any attention to the Supreme Court or not... AnonMoos (talk) 23:48, 14 April 2009 (UTC)[reply]
Where exactly did you read a scholar thinks the president has powers under the necessary and proper clause? That clause applies distinctly to Congress, article I, section 8. It does not give congress any affirmative powers (other than to enact the enumerated powers), and it particularly does not give the president any right to pass laws. The vast majority of executive power, even after Bush's assertions of expanding power, are directly pursuant to congressional laws that granted those powers. And as other posts point out, even the president is bound by the constitution Shadowjams (talk) 00:25, 15 April 2009 (UTC)[reply]
Your comments are indented as if they were in reply to my previous comment, yet they have no relevance to anything I said. AnonMoos (talk) 03:22, 15 April 2009 (UTC)[reply]

See Cornell Law School's annotated U.S. Constitution, Article II. I erred in using the term "necessary and proper," instead I should have used synonyms for necessary and proper. Clearly, the president has power to perform executive functions. My comment addressed the issue that the few cases that do exist often use different theories to arrive at a result. Few cases have been heard because of the political question doctrine. Many cases date from the 1800's before the rise of a powerful military and the regulation state. No one is asserting that the president can perform extraconstitutional functions. How is constitutionality determined? These cases are fascinating precisely because they address areas that were not addressed formally by the Constitution or the founders left actual practice to determine some aspects. Many would say Bush's actions were extraconstitutional. 75Janice (talk) 03:43, 15 April 2009 (UTC)75Janice[reply]

In theory the Supreme Court could undo over-reaching Presidential orders. In past practice, the Supreme Court has been very careful not to issue any rulings which a President might reject and refuse to enforce with the Justice Department and the armed forces, all controlled by the Executive Branch, headed by the President. Thus they would never have ruled that the Vietnam War was unconstitutional due to the lack of the required declaration of war by Congress. Humorist Finley Peter Dunne(1867-1936) had his fictional character Mr. Dooley say "...the Supreme Court follows the election returns." Taney was an exception to this, in attempting to enforce the clear language of the Constitution against the dictatorial decrees of Lincoln. Taney expected that Lincoln would have him locked up, but followed the dictates of the Constitution with regard to opposing Lincoln having his opponents locked up without due process. Edison (talk) 05:02, 15 April 2009 (UTC)[reply]
It can be argued that something like what the questioner has proposed has happened. In 1933 FDR banned the ownership of gold during the Great Depression. This ban wasn't repealed until the 70's. Today there are some people who fear the order will return. For details see Executive Order 6102. - Thanks, Hoshie 01:18, 16 April 2009 (UTC)[reply]
That is incredible! How did he justify that hoarding gold was enforceable under the trading with the enemy act of 1917? There was no enemy or trade 65.121.141.34 (talk) 13:46, 17 April 2009 (UTC)[reply]

Portugese traders in the 16th and 17th centuries Japan

How many Portuguese traders went to Japan in the 16th and 17th centuries? Please provide me with reference website links. Sonic99 (talk) 16:27, 14 April 2009 (UTC)[reply]

I googled "Japan Portugal trade" and got some useful looking links. This book looks as though it might be helpful. Karenjc 18:43, 14 April 2009 (UTC)[reply]

Public bathroom etiquette

I'm often surprised by what goes on in public washrooms and left wondering if there have ever been studies into the psychology of using public restrooms. Is it a common theme to not flush the toilet after using it(this one boggles my mind)? Is it rude to use the urnial next to someone else, even in crowded circumstance? or generally preferred to use a stall rather than a urinal when number one is called for? 142.132.4.45 (talk) 23:54, 14 April 2009 (UTC)[reply]

I know there is a lot of discussion of the unspoken (well, now more spoken) rules for men's toilet use, including rules like "no talking", "never pee next to someone unless you have to", "eyes front" and guidelines as to covering one's parts from view. I don't know of any studies to see what is actually done. I don't think I'd like to be that person, but I'd be interested in the results. Other things that might be relevant are mobile phones (answering a call is generally a no-no, but what about messaging?), flushing urinals ("If it's yellow, should it mellow?"), hand washing (and techniques), managing spills, etc. It's a lot more complex than it seems at first, and there's a lot of room for fauxs pas. Steewi (talk) 02:09, 15 April 2009 (UTC)[reply]
There's the courtesy flush too, which many folks never learn. 71.72.148.80 (talk) 03:00, 15 April 2009 (UTC)[reply]
Light a match. Kittybrewster ā˜Ž 19:57, 15 April 2009 (UTC)[reply]
Doesn't actually work. ā€” The Hand That Feeds You:Bite 20:33, 15 April 2009 (UTC)[reply]
One more thing that complicates it, is the general rule (similar to 'eyes front' and 'no talking') of ignoring other people in the room (or at least trying one's best to). This is possibly why some people never learn, because they are not observing what other people are doing. For example, if the guy next to you did, in fact, spill, you wouldn't notice it until he either tries to clean it up or goes out of the room. If you do see him cleaning it up, you are not going to watch how he does it, rather saving yourself (and him) any embarrassment by finishing up as quickly as possible. However, if he does walk out and just leaves his mess, you'll hate him forever, but you won't go after him to remind him.--KageTora (talk) 04:03, 15 April 2009 (UTC)[reply]
Googling for urinal etiquette brings up a host of interesting results. one might say a "shitload" of results Dismas|(talk) 04:27, 15 April 2009 (UTC)[reply]
How the other half lives, indeed! BrainyBabe (talk) 08:09, 15 April 2009 (UTC)[reply]

Re your first question, it's definitely been studied - from all kinds of angles - though the most common seems to be re handwashing behaviour, rather than the particular things you asked about. See [13]. Best, WikiJedits (talk) 11:45, 15 April 2009 (UTC)[reply]


Most of the replies have assumed (based on your mention of urinals) that you are mostly interested in the rules for male facilities. Is this okay, or would you like other contexts as well? 217.43.141.59 (talk) 15:37, 16 April 2009 (UTC)[reply]

My main concern has been for the male side of things as that's my experience, but the female side would be an interesting study. I assume they go in pairs so that they can talk privately with each other, besides that I wouldn't know of any odd happenings that go on behind those closed doors. ā€”Preceding unsigned comment added by 142.132.4.33 (talk) 00:30, 18 April 2009 (UTC)[reply]

April 15

Sindhi new year

Do Sindhi people have their own new year celebration? ā€”Preceding unsigned comment added by 99.226.95.73 (talk) 01:23, 15 April 2009 (UTC)[reply]

A little Google searching turns up "Cheti Chand", celebrated on the second day of Chaitra... AnonMoos (talk) 03:19, 15 April 2009 (UTC)[reply]

Goethe Quotations in "The Edge of Heaven"

One of the main characters in The Edge of Heaven is a literature professor, and at one of his lectures he attributes the following quotations to Goethe:

  1. "Wer wollte schon eine Rose im tiefsten Winter blĆ¼hen sehen? Alles hat doch seine Zeit: BlƤtter, Knospen, BlĆ¼ten... Nur der Thor verlangt nach diesem unzeitgemƤƟen Rausch."
  2. "Ich bin gegen Revolutionen, denn es geht genauso viel bewƤhrtes Altes kaputt wie gutes Neues geschaffen wird."

I thought these quotations were interesting, so I tried to track them down to read them in their original context, but I couldn't find their source. Many of Goethe's most important works are available on Project Gutenberg, and I searched many of them which seemed likely candidates, but I couldn't find the quotations. My local library has a large collection of works by and about Goethe, but I keep running into dead ends and can't locate the quotations. I am surprised that this information has been so difficult to find, considering the great success of the film.

I would greatly appreciate any help in finding the source of these quotations, either in the original German or in English translation. Thanks. --TantalumTelluride 05:20, 15 April 2009 (UTC)[reply]

One lead: Found both quotes together (?!) on this page, which looks like a conference paper by a Helge Martens. There is a contact name and address for the conference ([14]) - you could write and ask for the full paper with the citations/bibliography, or for how to contact Prof. Martens to ask. (The online version cites simply "Friedenthal" which I'm guessing could be Goethe: his life and times by Richard Friedenthal which should be in a lot of libraries.) Best, WikiJedits (talk) 11:39, 15 April 2009 (UTC)[reply]

"GC" in a date?

I have a document here, where a date is given as 24/6/1999GC. What could GC stand for in this circumstance? The document is from a Somali civil authority or court of law. E.G. (talk) 07:52, 15 April 2009 (UTC)[reply]

Gregorian calendar? BrainyBabe (talk) 08:11, 15 April 2009 (UTC)[reply]
Do we get to make jokes about Somali civil authority? ā€”Tamfang (talk) 17:42, 15 April 2009 (UTC)[reply]
Well, we could ask if this is a genuine court document or if E.G. has received a "Nigerian spam" message and is thinking it's real. (Nothing personal, E.G.; I don't know you.) Those messages often contain very peculiar usages and sometimes there's no guessing what they mean. --Anonymous, 18:27 UTC, April 15, 2009. 208.76.104.133 18:28, 15 April 2009
It was given by an applicant for a service at the office where I work, to prove the applicant's identity, so it is nowhere near any Nigerian spam. Gregorian calendar seems like a plausible explanation, thanks Brainybabe! E.G. (talk) 06:19, 16 April 2009 (UTC)[reply]
Our article Public holidays in Somalia states that the Gregorian calendar is used for official dates whilst the Islamic calendar pertains to religious dates, so BB would seem to be right. --Cookatoo.ergo.ZooM (talk) 10:07, 16 April 2009 (UTC)[reply]

Spring has sprung, the grass has riz

Spring has sprung, the grass has riz (or Spring has sprung, the grass is riz). Yahoo answers suggests that this was originally by Ogden Nash. I can't find a definitive source and date of authorship for this poem. Anyone here know? -- SGBailey (talk) 10:37, 15 April 2009 (UTC)[reply]

Ogden Nash is kind of a quotation sink for all authorless bits of comic doggerel just as Churchill gets all spare political quotes and Wilde gets arch put downs. These are the earliest cites for the opening lines I can find [15]. It was also used by Burma Shave which probably gave it wider currency, but not until 1951 apparently [16] meltBanana 12:44, 15 April 2009 (UTC)[reply]
Ta -- SGBailey (talk) 14:14, 15 April 2009 (UTC)[reply]
"Spring has sprung, the grass is riz, where last year's careless drivers is." -Burma Shave. SDY (talk) 14:50, 15 April 2009 (UTC)[reply]

How many independent countries are there in the world?

yes, it's homework but I can assure you that I don't know what to do... thousand sources and some say 190, others 195, help? --201.254.88.136 (talk) 19:15, 15 April 2009 (UTC)[reply]

Somewhere between 193 and 203. See list of sovereign states#Criteria for inclusion for details. 190 is definitely out of date. Algebraist 19:19, 15 April 2009 (UTC)[reply]

Thank you Algebraist, thank you very much. --201.254.88.136 (talk) 20:15, 15 April 2009 (UTC)[reply]

how do the amish make money?

do they even need money? after watching witness earlier these questions were raised, because arent they self contained? my theory was tourism, but they try to distance themselves, dont they? so i t doesn't all add up. 92.23.198.60 (talk) 19:50, 15 April 2009 (UTC)[reply]

Well, the Amish like most religions are of a varying spectrum. There are very traditional Amish who are extremely strict. These are generally the ones that seem to appear on the media. There are also some that are more open to using things like cars and electricity at a low level. Materialism is not a major priority for the Amish, so money would also be less important. That being said, they do need money to purchase the things that they can not produce, like land. Tourism does provide a good deal of income for the Amish groups who are involved with that. Also, things that the Amish can produce, such as furniture, quilts, and livestock can be sold to provide funds as well. The Amish do have the appearance of distancing themselves from the rest of society, but that does not mean they can not interact at all.65.121.141.34 (talk) 20:11, 15 April 2009 (UTC)[reply]
The thing to keep in mind is that the Amish are attempting to live their lives in a manner consistent with a simpler life, not go back to cave-man days. Money has been around for a good long time and they're just fine with using it, though as Mr. 34 says, their religion implicitly doesn't put a huge emphasis on those kinds of materialist things. Matt Deres (talk) 20:27, 15 April 2009 (UTC)[reply]
If you think about the "distinctive" Amish who drive the horse buggies then the primary source of income for them is almost always farming. There are plenty of secondary sources as well. Many will repair small motors; many will sell their farm produce direct to anyone who drops by, or at local farmers markets, and also quilts, firewood and similar.
For those nearer the mainstream there are plenty of jobs. I know some who run car body shops, and others who run guesthouses or restaurants. DJ Clayworth (talk) 20:48, 15 April 2009 (UTC)[reply]
Travelling through Amish country last summer, I was struck by the extensive range of Amish handicrafts for sale from converted barns by the sides of major roads. I guess that this is probably a significant source of secondary income. AlexTiefling (talk) 21:00, 15 April 2009 (UTC)[reply]
In addition to the sale of farm goods and handicrafts, Amish men are often excellently skilled labourers, and can earn a significant amount of money as builders, carpenters, gardeners, etc. They are known for their reliability and work ethic and are implicitly considered to be honest, so those who want a good job done will often pay above-market price to have the Amish extend their houses, make their kitchen cupboards, seed their lawn, look after their horses, etc. It is a common perception by Americans who live in "Amish country" that the Amish are actually very well-off financially. They spend little of what they earn and save the rest, usually for rainy days, dowries, and to assist their (often numerous) children in the future. They also have significant equity in the amount of land they own, handed down from generation to generation. Maedin\talk 21:19, 15 April 2009 (UTC)[reply]
I consistently see advertisements for Amish furniture, which is, I gather, usually higher quality and more expensive then other furniture. I have also heard that recently some of the Amish, while usually sheltered from economic vagaries, are having financial difficulties due to the recent economic downturn [17] (less people buying souvenirs and the like). -- 128.104.112.117 (talk) 21:41, 15 April 2009 (UTC)[reply]
Think: how did people earn money before the industrial revolution? The answer would usually be farming, simple manufacturing, and pre-industrial service industries. --PalaceGuard008 (Talk) 23:23, 15 April 2009 (UTC)[reply]
The Amish aren't hermits. They are aware of the rest of the world, and freely interact with it. They just choose to live simply themselves. Quite frankly, the Amish lifestyle is certainly cheaper. They grow a lot of their own food, and support themselves otherwise with selling handicrafts and the like. Imagine if you didn't need to buy TVs and make car payments and pay for electricity and make your own clothes. It's probably a lot easier to live on a smaller income when you don't have to spend much anyways! --Jayron32.talk.contribs 02:04, 16 April 2009 (UTC)[reply]
There are also Amish-run restaurants. They do use electricity. I think they justify this because the electricity is for the customers, not for themselves. I've also seen Amish food in regular grocery stores, like Amish potato salad and Amish baked beans. In some cases these are Amish-made, in other cases someone just stole their recipe. (I wonder about the legality of selling something as "Amish" which isn't Amish-made.) StuRat (talk) 12:44, 16 April 2009 (UTC)[reply]
Is "Amish" a badge of origin or a trademark? --PalaceGuard008 (Talk) 13:14, 16 April 2009 (UTC)[reply]
I believe the article you are looking for is Appellation, per the red link... --Jayron32.talk.contribs 01:40, 17 April 2009 (UTC)[reply]
Hmm, sounds like time for a new article. --PalaceGuard008 (Talk) 03:33, 17 April 2009 (UTC)[reply]

Wage-earning in colonial Spanish America

Any details on the development of wage-labor within the poor that slowly replaced slavery in the Spanish colonies? How it formed and to what extent? Grsz11 20:01, 15 April 2009 (UTC)[reply]

From a history of North America class, I vaguely remember a trend starting in sugar cane processing facilities.NByz (talk) 06:06, 16 April 2009 (UTC)[reply]

Thatcher's Childen

A UK-centric one, this. I have heard of chavs referred to as Thatcher's children. Anyone have a social or economic explanation for this? I have read the Wiktionary definition, but this would seem to apply to slightly more affluent kids of that generation. I am speaking of a lower rung of the social scale. Thanks FreeMorpheme (talk) 20:17, 15 April 2009 (UTC)[reply]

It would be worth reading Thatcherism. Thatcher's strong policies revived the British economy, but at the cost of massive unemployment and increase in poverty levels - the effects of which are still felt today, and are possibly demonstrated by the chavs. You can find a good summary by the BBC at the end of the Thatcherism article: To her supporters, she was a revolutionary figure who transformed Britain's stagnant economy, tamed the unions and re-established the country as a world power. Together with US presidents Reagan and Bush, she helped bring about the end of the Cold War. But her 11-year premiership was also marked by social unrest, industrial strife and high unemployment. Her critics claim British society is still feeling the effect of her divisive economic policies and the culture of greed and selfishness they allegedly promoted. So the comment you note about the chavs means something like "these are the type of people created by Thatcher's policies, the inevitable result of generational unemployment and poverty". Gwinva (talk) 20:29, 15 April 2009 (UTC)[reply]
Note also the play Thatcher's Women by Kay Adshead. The eponymous women are thrown out of work in a canning factory in the North West, and go to London to try to make a living as prostitutes. I'm pretty sure that the title was not coined just for this play. --ColinFine (talk) 23:13, 15 April 2009 (UTC)[reply]

Japan and Germany

What do Japan and Germany have in common that made them such economic powerhouses? (apart from being on the floor in 1945) - thanks for info. --AlexSuricata (talk) 21:13, 15 April 2009 (UTC)[reply]

To be succinct: Post-war economic aid.--KageTora (talk) 01:43, 16 April 2009 (UTC)[reply]
Also, they have large populations, and the infrastructure and expertise for industrialization was in place after World War II. Being that they were advanced economies before the war, and during the war, it's not hard to see how, with a small push from the outside, their economy could be restarted within a few decades. It wasn't as though the countries were leveled during the war. It was just a matter of taking the industry in place, and retooling it for peacetime economy. Of more interest is the difference of what happened between West Germany and East Germany in the decades following the war. --Jayron32.talk.contribs 01:58, 16 April 2009 (UTC)[reply]
As for Japan, see our article. Korean war boosted the economy. Additionally, because of the Article 9 of the Japanese Constitution, the military budget of Japan was small. So the government could invest in infrastructure. Oda Mari (talk) 07:28, 16 April 2009 (UTC)[reply]

Japan started with the kinds of sweatshops that China has, or maybe had especially a few years ago. It built itself up on sweat - and also started as crappy manufacturers, like plastic chinese crap. 94.27.231.11 (talk) 07:52, 16 April 2009 (UTC)[reply]

Regarding economic aid, it should be noted that Japan was not a participant of the Marshall Plan. According to Japanese post-war economic miracle, Japan's recovery was due to the Korean War and government intervention. A Quest For Knowledge (talk) 16:23, 16 April 2009 (UTC)[reply]
Reading the article, to clarify, that's intervention as in economic interventionism. - Jarry1250 (t, c) 19:04, 16 April 2009 (UTC)[reply]
I think the main factor was that they were both prohibited from major rearmament spending so, unlike their major economic competitors (UK, U.S.A., Soviet Union, France, etc) their national wealth was invested in manufacturing capacity and used to generate additional capital rather than being spent on non-productive highly expensive projects like atomic bombs and intercontinental ballistic missiles. AllanHainey (talk) 17:20, 17 April 2009 (UTC)[reply]
Both have excellent education systems (don't know before late 20th century). Both have small militaries and spend less on defence than other first world countries. Japanese save their money freeing up money for investment. Japan invests in factories in China to produce Japanese goods cheaply for export and domestic consumption. Both are hard-working peoples with a work ethic. Germany was scientifically advanced before WWII.
Sleigh (talk) 05:07, 18 April 2009 (UTC)[reply]
To Sleigh. After the war, Japan had no diplomatic relations with People's Republic of China until 1972. See Joint CommuniquƩ of the Government of Japan and the Government of the People's Republic of China. So the Japanese investment in factories in China has nothing to do with the postwar economic recovery. Oda Mari (talk) 05:40, 18 April 2009 (UTC)[reply]

April 16

police secrets -- so effective it's censored even in the report?

Were there cases (though obviously we have found out since) where police decided to hide, even from other police officers, (ie in their police reports etc), some particular details of a novel crime method, when the new method is incredibly effective, very interesting and newsworthy (so that the other police would be sure to gossip about it, and it would surely get out) and most importantly, the method is by its nature impossible to guard against. Thanks! 94.27.231.11 (talk) 09:02, 16 April 2009 (UTC)[reply]

If this hypothetical criminal method is impossible to guard against, how did the police figure it out and make a police report about it? Livewireo (talk) 13:03, 16 April 2009 (UTC)[reply]

Whilst not perfectly related to this - there are regularly discussions in the media about whether reporting 'how' things are done is a good idea or not. For instance Ben Goldacre notes in his recent article (http://www.guardian.co.uk/commentisfree/2009/mar/28/media-reporting-suicide-bad-science) that there is good cause to be careful when it comes to reporting Suicide - it seems that if something is reported you'll see a spike in that particularly form of suicide. E.g. His article notes that in Hong Kong in 1998 the media reported a case of carbon-monoxide suicide which had a very specific method used - in the 10 months prior to the media-reports there were no suicides of this type. Over the next year there were 40. Obviously there's more questions to ask of this than pure numbers (is is that those would be committing suicide are just using a prominent-in-their-mind method, or is it an actual increase in the number of suicides? Either is a cause for concern but would be important to understand the issue further). ny156uk (talk) 16:35, 16 April 2009 (UTC)[reply]

I read one interview with a pharmacist who bragged that he knew 10 untraceable murderous poisons that would be lethal to humans, ideal for committing the perfect murder; and then coyly stated it would be unethical to blab. Tempshill (talk) 19:53, 16 April 2009 (UTC)[reply]
If you are going to murder someone like that, toxins are far better then poisons, especially as toxins can be murder to trace. 65.121.141.34 (talk) 20:25, 16 April 2009 (UTC)[reply]
I don't know about domestic police, but the FBI and CIA have many "sources and methods" that are considered so secret that almost nobody was told about them. Even President Truman was not cleared to know about the Venona decrypts, for example. --98.217.14.211 (talk) 21:31, 16 April 2009 (UTC)[reply]
There is a related topic (subject of debate) in computer security called full disclosure. 66.127.52.118 (talk) 09:18, 19 April 2009 (UTC)[reply]

Revealing a Psychopath

Correct me if I'm wrong, but one of the traits of a psychopath is having little to no emotion/empathy and being a very good liar. Are there any questions that you can ask a psychopath that would "give away" his/her 'psychopathness'? --Reticuli88 (talk) 13:13, 16 April 2009 (UTC)[reply]

There are a number of pop-psychology attempts at doing this kind of thing. I'm positive Snopes did a piece on one of these so-called psychopath detector questions. In reality, no, there's no simple way to identify people like that, at least with any kind of consistency. Besides all the usual caveats about the complexity of people and the general unreliability of simple tests, there's the basic issue that psychopaths are either good at covering their tendencies or already in custody. It's a little like asking if there are any simple questions you could ask an expert liar and reliably identify them as such. Matt Deres (talk) 13:24, 16 April 2009 (UTC)[reply]

Do you have a link for that Snopes article? --Reticuli88 (talk) 13:29, 16 April 2009 (UTC)[reply]

I think it's probably this one. - EronTalk 13:31, 16 April 2009 (UTC)[reply]
You can simply give the subject a Voight-Kampff Empathy Test. See here [18] for the startling results of such a test.... 161.181.53.10 (talk) 02:46, 17 April 2009 (UTC)[reply]

Green Hindu figure

While in Amsterdam this week, my wife took a picture of three figures standing on separate rostrums in the middle of what appears to be a square or marketplace of some sort. And they seem to have tip jars in front of them like it was some sort of performance piece. One is Darth Vader, another is Batman, and the last is a green figure. She only sent pictures of the first two due to my geekier tendenciesĀ :-) but I'm curious who the third is. She described it as a "Hindu guy". I'd post an image but I doubt it would do any good. He's standing in the far background and is out of focus. So, he's green, seems to have some sort of head-dress, is holding a staff, and her description of "Hindu" might not be completely accurate. Anyone know who this might be? Any Amsterdamers here who might have seen the people in question? Dismas|(talk) 14:54, 16 April 2009 (UTC)[reply]

I wouldn't spend too much time in trying to identify who the person was pretending to be. Touristy European cities are filled with people who dress up in strange costumes and make-up, pretend to be statues and pose for photographs in exchange for small donations. Some of the disguises are based on historical or well-known figures, but many are simply a result of the actor's imagination and materials available to him or her. --Xuxl (talk) 15:24, 16 April 2009 (UTC)[reply]
Green seems like the wrong colour ā€“ blue would make more sense for a "Hindu" figure, because Lord Vishnu and his avataras Lord Krishna and Lord Rama are generally portrayed with blue skin. Vishnu and Krishna wear hat-like crowns in a great deal of art. Rama is sometimes shown with the ascetic's matted topknot in his hair. None of the three carries a staff, but Vishnu has a mace (also a conch shell and a discus) and Rama a bow; Krishna would also have peacock feathers somewhere. So Rama is one guess, though if "Hindu" is wrong none of this would applyĀ :) Best, WikiJedits (talk) 15:50, 16 April 2009 (UTC)[reply]
There is also the Green Tārā, but she's a female Buddha or Bodhisattva, not a Hindu guy. ---Sluzzelin talk 20:11, 16 April 2009 (UTC)[reply]
There is Shiva who uses a trident, not a staff. Jay (talk) 12:09, 17 April 2009 (UTC)[reply]
For green "Hindu" superheroes, there is Nagraj from "The Home Of Indian Superhero Comics". ---Sluzzelin talk 12:32, 17 April 2009 (UTC)[reply]

First cousins once removed's child

It doesn't explicitly state the name for that relation in the cousin article, and it gets pretty complicated. What would the child of my first cousin once removed be to me? hmwithĻ„ 18:17, 16 April 2009 (UTC)[reply]

According to the chart in consanguinity, it would be your first cousin twice removed. --Gadget850 (talk) 18:33, 16 April 2009 (UTC)[reply]
Hm, that still doesn't make sense to me... haha. Somewhere else online (not a necessarily reliable place), someone interpreted the situation as we'd be second cousins twice removed. It's the child of my parent's first cousin. hmwithĻ„ 18:55, 16 April 2009 (UTC)[reply]
The person you speak of, the grandchild of your first cousin, is your first cousin twice removed. Your grandchild would be that person's third cousin. Except in some backwoods regions, or among royals and nobility, where they might also be additional degrees of cousinship. Edison (talk) 19:09, 16 April 2009 (UTC)[reply]
Okay, I think I made a mistake in explaining the situation. My grandmother is the sister of this person's grandmother, so this person isn't the grandchild of my first cousin. However, I am the grandchild of this person's great-aunt. hmwithĻ„
Haha, maybe I'll just give up. I'm more confused now than before. hmwithĻ„ 19:29, 16 April 2009 (UTC)[reply]
That means that you're second cousins. I presume that by 'First cousins once removed's child', you mean that you and your cousin are each the children of people who were themselves first cousins. AlexTiefling (talk) 19:38, 16 April 2009 (UTC)[reply]
Ah, yes. That's what I mean. We're just plain & simple second cousins? Then what am I to my father's cousin, my second cousin's mother? hmwithĻ„ 19:54, 16 April 2009 (UTC)[reply]
No, there's something weird there. If your grandmother and this person's grandmother were sisters, that makes you and this person second cousins, and their child is your second cousin once removed. But if you're the grandchild of this person's great-aunt, that makes you second cousins twice removed, and the child of this person is your second cousin, three times removed. It can't be both. -- JackofOz ( talk) 19:59, 16 April 2009 (UTC)[reply]
How did you get at that? If the OP and this person have grandmothers who are sisters then the OP's grandmother is the person's great aunt. A second cousin twice removed is a second cousin's grandchild or a grandparent's second cousin. Do I have it wrong? Zain Ebrahim (talk) 13:07, 18 April 2009 (UTC)[reply]
The last bit (grandparent's second cousin) is wrong. Kittybrewster ā˜Ž 13:38, 18 April 2009 (UTC)[reply]
No, it's not. The relationship works both ways so your comment below is wrong. See cousin. Zain Ebrahim (talk) 14:33, 18 April 2009 (UTC)[reply]
Quote please Kittybrewster ā˜Ž 14:47, 18 April 2009 (UTC)[reply]
The article actually specifically states what I said:
"My second cousin's grandchild and I are second cousins twice removed"
and
"My grandparent's second cousin and I are second cousins twice removed" Zain Ebrahim (talk) 14:51, 18 April 2009 (UTC)[reply]
What you said is that where A has 2 children B and C, where B has grandchild D and D has a grandchild E and where C has grandchild F, you say C is 2nd cousin to E? Not so. They are great great great uncle and great great great nephew. Kittybrewster ā˜Ž 15:17, 18 April 2009 (UTC)[reply]
I didn't say that at all. Read what I said then read those two extracts from the article. They're both the same. I basically explained the two types of "second cousin twice removed" - you say that the one type is wrong. But it's not. Consider the relationship between E and F both ways. Zain Ebrahim (talk) 15:35, 18 April 2009 (UTC)[reply]
I mistook you. Sorry. My contention is that F says "E is my 2c2r" while E says "I am F's 2c2r". Kittybrewster ā˜Ž 17:54, 18 April 2009 (UTC)[reply]
But there's nothing wrong with that. According to cousin, E is F's 2c2r and F is E's 2c2r. Zain Ebrahim (talk) 18:05, 18 April 2009 (UTC)[reply]
I am disagreeing with you on the detail. I am JackOfOz's British friend (or at least, I would like to think so). He is not my British friend (because he is upside down). My uncle is not my nephew. I contend a relationship is "attached" at one end. Kittybrewster ā˜Ž 18:10, 18 April 2009 (UTC)[reply]
Well, the article disagrees with you. For cousins (and removed cousins), the relationship is not attached at any end. Do you have a source saying otherwise? Zain Ebrahim (talk) 18:14, 18 April 2009 (UTC)[reply]
JackofOz, I don't want the child of that person. I want who that person would be. I want the child of my great aunt's child, who is the grandchild of my great aunt. I don't want anymore generations down from that; I don't need the granchild's child. hmwithĻ„ 17:22, 18 April 2009 (UTC)[reply]
Second cousin. Kittybrewster ā˜Ž 17:54, 18 April 2009 (UTC)[reply]
OK. Clear now. I've struck out my comment below. -- JackofOz (talk) 21:25, 18 April 2009 (UTC)[reply]
It might help to think of the "removed bit" by itself. It means one generation's difference from you, either older or younger. Your father's first cousin is your first cousin, once removed. That person's daughter is your second cousin (no removal, because you are at the same generation). You have great-grandparents in common, because your grandparents were siblings. BrainyBabe (talk) 19:56, 16 April 2009 (UTC)[reply]
Ah, okay. You finally made it clear to me. Thanks everyone for your help! I know I was a bit confusing.Ā :) hmwithĻ„ 20:28, 16 April 2009 (UTC)[reply]

Okay, now that we've cleared that up, let me just point out that the question as originally asked was ambiguous. If A and B are first cousins once removed, they are one generation apart either way. A is B's first cousin once removed but B is also A's first cousin once removed. So "the child of my first cousin once removed" is "my first cousin twice removed", as Gadget said originally, if we're talking about cousins of a later generation than mine. But if I'm the one in the later generation, then "the child of my first cousin twice removed" is "my second cousin", as Babe said. --Anonymous, edited 00:39 UTC, April 17, 2009.

There is a chart in consanguinity that should be of help. --Gadget850 (talk) 00:46, 17 April 2009 (UTC)[reply]
If the chart is confusing to some, then try this formula. Find the nearest common ancestor to both of you. The shortest distance of that ancestor to either of you determines the COUSIN NUMBER, and the difference in number of generations between the two of you is the REMOVAL NUMBER. Thus, if you and a relative share an ancestor who is your great-grandmother, and her great-great-great-great-grandmother, you are second cousins (because the CLOSEST is a great-grand relationship) who are three-times removed (because there are three generations difference between you).--Jayron32.talk.contribs 04:06, 17 April 2009 (UTC)[reply]
True. When you have the shortest distance, you count the "G"s. So if the shortest distance is Great Grandparents then they will be 2nd cousins. Plus (whatever) removed. Kittybrewster ā˜Ž 13:17, 18 April 2009 (UTC)[reply]
But we haven't cleared it up. See my post above. The OP has provided conflicting information, and at most one of the two things he said about this relationship can be true. -- JackofOz (talk) 12:59, 18 April 2009 (UTC)[reply]
I think we have. OP is 1st cousin once removed of OP's 2nd cousin's mother. That is not the same thing as OP's 2nd cousin's mother is OP's 1st cousin once removed. Kittybrewster ā˜Ž 13:27, 18 April 2009 (UTC)[reply]
See http://genealogy.about.com/library/nrelationshipchart.htm. -- Wavelength (talk) 18:36, 18 April 2009 (UTC)[reply]
That is one-sided and doesn't show my uncle/aunt. Kittybrewster ā˜Ž 20:45, 18 April 2009 (UTC)[reply]
So? It still shows that your comment (3 posts up) is wrong as I explained above. Start with a common ancestor's great grandchild (the OP) and grandchild (OP's 2nd cousin's mother) - the common ancestor needs to be their nearest common ancestor. The table then shows that the OP and the OP's 2nd cousin's mother are each other's first cousin once removed. Zain Ebrahim (talk) 00:52, 19 April 2009 (UTC)[reply]
It also shows my uncle is my nephew - which he isn't. Kittybrewster ā˜Ž 08:25, 19 April 2009 (UTC)[reply]

Afghanistan History between 1983 and 2000

Hi I have to give a Presentation on the history of conflicts in afghanistan concentrating mainly on the civil war and the time period of 1983-2000.

Could you please provide me with the major events between this time period and also anything you think would be of interest for such a presentation.


Many Thanks. ā€”Preceding unsigned comment added by 94.197.183.245 (talk) 20:52, 16 April 2009 (UTC)[reply]

It is the policy of the Reference Desk not to do your homework for you. However we have an article History of Afghanistan which contains much helpful information and links to other articles. DJ Clayworth (talk) 21:20, 16 April 2009 (UTC)[reply]

Hi, its not my homework, its research for my presentation.

Many Thanks ā€”Preceding unsigned comment added by 94.197.183.245 (talk) 21:33, 16 April 2009 (UTC)[reply]

I'm curious about your presentation. 1983-2000 seems like a very arbitrary set of dates, beginning in the middle of the Soviet war, and ending before the post-9/11 invasion. Perhaps if reading Republic of Afghanistan and History of Afghanistan since 1992 don't help, you can give us some more detail on what you're looking for. --Sean 21:57, 16 April 2009 (UTC)[reply]


It's to do woith work, I'm in the forces. We've split into 4 groups, two groups are pre 1983, in seperate time periods my grouping is 1983-2000 and then the final grouping is 2000 to modern day conflict. ā€”Preceding unsigned comment added by 94.196.192.71 (talk) 23:03, 16 April 2009 (UTC)[reply]

So, its work...that you are doing from home. Please read the articles that were given earlier, they have some excellent sources. Livewireo (talk) 13:15, 17 April 2009 (UTC)[reply]
The reason we don't do homework for you (whether it's for school or for your job) is that the point of these tasks is to get you to go and find things out for yourself. There is no benefit to you if your presentation consists of you repeating things other people told you. DJ Clayworth (talk) 13:19, 17 April 2009 (UTC)[reply]

How Many people are evangelical?

I'm not sure I understood correctly Evangelicalism#Global demographics. Are there 420 million evangelical Christians? 22:43, 16 April 2009 (UTC) ā€”Preceding unsigned comment added by 217.132.172.12 (talk)

Evangelical christians is a broad umbrella term generally used to refer to christians who self-identify as actively working to convert others to Christianity. Given that there are roughly 1.637-1.923 billion Christians in the world (see List of religious populations), using that range, 420/1627 gives 25.8% and 420/1923 gives 21.8%; so roughly 1/5th to 1/4th or so of christians are evangelical. I do not find that number hard to believe. --Jayron32.talk.contribs 01:49, 17 April 2009 (UTC)[reply]

April 17

Taxation in the United States

I'm looking for information on where the money for the federal budget comes from. My understanding is that the majority comes from taxing the populace. What I'm looking for is, what percent of the US budget comes from what percent of the popuation. My suspicion is that a large portion of the money comes from a small portion of the people, but I have no data to back that up192.136.22.4 (talk) 00:20, 17 April 2009 (UTC)[reply]

This took me about 3 seconds to find using Google: [19] Is that what you are looking for? --Jayron32.talk.contribs 01:42, 17 April 2009 (UTC)[reply]
yes and no, It told me that 45% or so of the government's spending budget comes from income tax, but it doesn't say what percentage of the population that comes from, that is to say, i'm curious what the breakdown between the various income brackets. Thanks though Jayron192.136.22.4 (talk) 02:26, 17 April 2009 (UTC)[reply]
This took me an additional 60-70 seconds to find using google: [20]. Is that what you are looking for? --Jayron32.talk.contribs 04:04, 17 April 2009 (UTC)[reply]
Jayron32, note that (if I understand the question correctly), your second source is not really responsive, since it gives only percentages for the federal income tax, and not for federal revenues as a whole. It therefore overstates the contribution of large earners to federal revenues. John M Baker (talk) 20:02, 17 April 2009 (UTC)[reply]
If you know income tax contributes 45%, you can just adjust the percentages accordingly? --PalaceGuard008 (Talk) 03:20, 18 April 2009 (UTC)[reply]

Lebanese politics

Can someone tell me what is going on in Lebanon regarding its upcoming election in June? Here some videos about it. [and candidates in Tripoli], [Hariri and Syrians], [Lebanese Army in Bekaa Valley], [OTV: Al-Mustaqbal scandal about Shia and Sunni] and [Al-Mustaqbal thugs beating up Homentmenā€™s girls] ā€”Preceding unsigned comment added by 76.64.53.139 (talk) 01:42, 17 April 2009 (UTC)[reply]

Anyone knows of The History of the Nation?

Does anyone know about the work, The History of the Nation, possibly printed in the early half of the 20th century? Several paintings have been noted to be published in it.[21][22][23][24][25][26][27] However, I am unable to locate such a book on Worldcat or Google. Does anyone have information that pertain to this work? Jappalang (talk) 02:27, 17 April 2009 (UTC)[reply]

It appears the books in question would be "Hutchinson's Story of the British Peoples" and "Hutchinson's History of the Nations", multi-volume illustrated histories by a Walter Hutchinson. This Hutchinson fellow seems to be pretty much forgotten, unlike the illustrator, Richard Caton Woodville.--Rallette (talk) 07:22, 17 April 2009 (UTC)[reply]
There is an article on Henry Payne (artist),1868 - 1940, who seems to be the artist in question. Searching for The History of the Nation in sites dealing with rare / out of print books gives no results, however, there is Glorious Battles in English History by Major Wylly (and a few more), for which Harry Payne is listed as the illustrator. There is (see above) HutchinsonĀ“s Story of the British Nation, where Payne has been involved as an illustraor also, but this was published as a series of weekly softcovers. --Cookatoo.ergo.ZooM (talk) 07:48, 17 April 2009 (UTC) PS: This site [28] has an offer for volumes 3 to 28. --Cookatoo.ergo.ZooM (talk) 07:50, 17 April 2009 (UTC)[reply]
I did find a couple of Hutchinson's books, including one called "Story of the Nations" which may or may not be the same as "History of the Nations", on ZVAB -- Ferkelparade Ļ€ 11:09, 17 April 2009 (UTC)[reply]

Why red shirts?

I was in Thailand a little while ago, and the symbolism behind the yellow shirts is obvious (Yellow is the King's color by an old tradition of birthdays and days of the week). Why the red shirts? SDY (talk) 02:32, 17 April 2009 (UTC)[reply]

Just speculation, but the NUFDAD appears to be closely associated with the People's Power Party (Thailand). Our article there states that the party supports a "populist welfare" platform. When I read "populist welfare" and "People's ANYTHING" I think leftist/socialist/C-word and the traditional color associated with that ideology has usually been red. Other uses of the color red are often related to revolution in general, especially left-wing revolution, see Red flag. --Jayron32.talk.contribs 04:00, 17 April 2009 (UTC)[reply]
For pre-Socialist populist and progressive red shirts, see Garibaldi's camicie rosse.---Wetman (talk) 08:02, 17 April 2009 (UTC)[reply]
Maybe they want to extend their eligibility to compete in college athletics. Or maybe it is like the quarterback in American pro-football wearing a red jersey in practice as a signal that no one is allowed to tackle him. Edison (talk) 02:22, 18 April 2009 (UTC)[reply]
There's definitely another meaning of redshirt which would be embarrassingly inappropriate. There are some traditional (Chinese?) associations for red as well (bravery and weddings and whatnot) that are ingrained in a lot of Asian cultures, and I'm thinking that those would be more influential than the traditional Western associations (i.e. Garibaldi) and I'd be quite surprised to see Thaksin Shinawatra painted as a Communist, though that party has been absent in Thailand for some time (after being banned) so I doubt it'd be public if it were true. The historical Thai flag (Siamese, really), according to our article, was red, but the group in question isn't really a traditionalist group. SDY (talk) 18:33, 18 April 2009 (UTC)[reply]

English royal standard

Hi. I'm looking for a merchant who will sell me the old France-England royal standard flag, rather than the new royal standard, or the original three leopards alone. Does anybody here know of such a business? Maybe some theatre would sell one of these? Please be of some assistance. Thank you. Catterick (talk) 08:13, 17 April 2009 (UTC)[reply]

The flagshop [29] may be an option. --Cookatoo.ergo.ZooM (talk) 08:22, 17 April 2009 (UTC)[reply]

I don't mean to be rude...perhaps I should have specified that I am looking for these in flag form: Catterick (talk) 08:33, 17 April 2009 (UTC)[reply]

Mr Flag will do a beskpoke/custom flag if they don't already stock it. Nanonic (talk) 10:38, 17 April 2009 (UTC)[reply]
You will probably want to specify whether you want France Ancient (on the shield in the images shown) or France Modern (on the flag). AlexTiefling (talk) 11:57, 17 April 2009 (UTC)[reply]
Not leopards but lions.
The emblazon of the shield on the right looks wrong. I would emblazon seme of fleur-de-lis with less fleur-de-lis.
Sleigh (talk) 07:03, 18 April 2009 (UTC)[reply]
They are leopards, because multiple lions passant guardant are often described as leopards. To the best of my knowledge, there's no other heraldic leopard, and you'd probably have to use a special blazon to refer to a 'real' leopard, in much the same way as one blazons a Bengal tiger for the real thing, because an unqualified tiger in heraldry is a mythical beast. I've often seen seme de lis blazoned at about that density, but it may appear to be heavily scattered because it's effectively a reduced copy of France Ancient, rather than a quarter azure seme de lis or. AlexTiefling (talk) 18:31, 18 April 2009 (UTC)[reply]

St. George's Day

Why isn't St. George's Day celebrated in the States? Is it a Puritan, Yank Anglophobia, even though Yanks supposedly pride so much in their "pure Englishness"? After all, there are Highland festivals and St. Patrick's Day Parades, although admittedly much fewer signs of a Welsh presence. I don't know why. I've had a few heart-stopping moments when seeing the St. George flying over the Roanoke Colony museum and a few other historic places; even the old Union Jack without St. Patrick is a sight to see for sure. Catterick (talk) 08:23, 17 April 2009 (UTC)[reply]

You might start by asking why St. George's Day isn't celebrated in England, the country that he is the patron saint of. Since there are hundreds of patron saints that could be celebrated, the question is why St. Patrick is celebrated (he's the only one who is, as far as I know). One possible reason is that "Irish" is seen as a popular yet underprivileged ethnicity; another is that it's a day celebrated by drinking huge amounts of alcohol, and who doesn't want to be part of that? DJ Clayworth (talk) 13:15, 17 April 2009 (UTC)[reply]
Which definition of Yankee are you using? The ones in New England whose major city Boston is now the stereotypical center of Irish American? English American heritage is often considered too common to be distinctive (or for many, too ancient to be remembered) Rmhermen (talk) 14:08, 17 April 2009 (UTC)[reply]
I've lived in the States for most of my life and have never detected any "pride in pure Englishness". People will routinely inform you that they are "1/8th Cherokee" or whatever, with the other 7/8ths not worth mentioning. --Sean 13:55, 17 April 2009 (UTC)[reply]
The U.S. in infancy fought two wars with Britain, which may have diminished identification with England specifically on the part of many who were creating the very notion of being American. Add English-speaking immigrants whose experience with John Bull wasn't always positive (Scots, Irish). Some New Englanders do pride themselves on being Yankees (in the American, not the English, sense), but that would emphasize differences with England, since "Yankee Doodle" was originally derision. And many Irish-Americans, and Irishmen in general, would agree that it'd be great to see the Union Jack without St. Patrick's cross. Or, as the song goes, the harp without the crown. --- OtherDave (talk) 20:13, 17 April 2009 (UTC)[reply]
Call me a cynic, but I suspect the fact that the "traditional" celebration of St. Patrick's day involves drinking Guinness may have something to do with its popularity. --Tango (talk) 13:47, 18 April 2009 (UTC)[reply]

Cooking practices of the Tarahumara - do they bake, roast, or fry much?

According to this http://boards.msn.com/thread.aspx?threadid=882128 the Tamahumara, who live in remote desert parts of Mexico, never get cancer, although this could be an urban myth since they do not make the news for the world's oldest people. The article linked from that link suggests they mostly eat some sort of maize dish that is prepared in water. Do the Tarahumara roast bake or fry their food very much? If not, then they would be exposed to very little acrylamide, which may be carcinogenic, and this might be the reason perhaps. I have read both the acrylamide and the Tara humara articles. 78.146.249.32 (talk) 11:47, 17 April 2009 (UTC)[reply]

Actually, they get cancer all the time. You now have as much evidence that they do as that they don't. Extraordinary claims require extraordinary evidence, and the claim that a group of people are not subject to cancer, diabetes, and other common human ailments is certainly extraordinary, and requires better evidence than a random internet comment like the one I just invented to support my point, or the one "SkyHunter" probably just invented to support his. For what it's worth, a PubMed search for "cancer Tarahumara" gets no results and only 2 hits on Google. --Sean 14:08, 17 April 2009 (UTC)[reply]
And what do you mean by "the world's oldest people"? Says who? Who then was a gentleman? (talk) 03:57, 18 April 2009 (UTC)[reply]

The question is "Do the Tarahumara roast bake or fry their food very much?" 89.242.147.172 (talk) 10:46, 19 April 2009 (UTC)[reply]

Which is the best type of Gladiator?

If you were trasported back in time and forced to be a gladiator, which of the several types of gladiator would it be best to be regarding survival? 78.146.249.32 (talk) 12:54, 17 April 2009 (UTC)[reply]

The type that is also an emperor. LANTZYTALK 13:02, 17 April 2009 (UTC)[reply]
But Roman emperors had an extremely low survival rate, regardless of whether or not they were gladiators. They were always being stabbed or poisoned by their rivals. 209.251.196.62 (talk) 16:23, 17 April 2009 (UTC)[reply]
After reading List of Roman gladiator types, they are all either poorly armored or poorly armed. No choice has any real significant advantage over the other. That said, I agree with 209. Easy to be a gladiator when your competition is afraid to kill you! Livewireo (talk) 16:30, 17 April 2009 (UTC)[reply]
It wouldn't be much fun to watch if one type of gladiators always won! --PalaceGuard008 (Talk) 03:17, 18 April 2009 (UTC)[reply]
I think I'd prefer to be one of the Andabatae, on horse, against some guy on foot. Who then was a gentleman? (talk) 04:00, 18 April 2009 (UTC)[reply]
I'd rather be the kind that escaped the night before I had to fight... --Alinnisawest,Dalek Empress (extermination requests here) 22:15, 18 April 2009 (UTC)[reply]

Religious leaders who have converted

Some friends and I were discussing the possibility of a pope converting to Islam, whether it would automatically terminate his papacy, what effect it would have on the church, etc. It got me thinking: has anything of that sort ever happened in reality? Has a religious authority ever converted suddenly to another religion? I vaguely recall a medieval Catholic bishop who converted to Islam, but Google has been useless. LANTZYTALK 13:26, 17 April 2009 (UTC)[reply]

The Pope, or any other Roman Catholic, would be excommunicated lata sententia for apostasy. I don't know if any medieval bishops converted to Islam but certainly many regular Catholics did, wherever there was Christian-Muslim contact (Spain, Sicily, Syria). Muslims often converted to Christianity too, and both converted to Judaism and vice versa. I think it happened a number of times that a Catholic priest would start using the Greek rite, but that's not exactly conversion and not exactly apostasy. It happened somewhat regularly whenever crusaders conquered Greek territory. In any case, the conversion of the Pope to Islam is exceedingly unlikely... Adam Bishop (talk) 14:07, 17 April 2009 (UTC)[reply]
Of course, lots of people have converted between the various strands of Christianity. The most notable of these (outside the Reformation period) might well be John Henry Newman, who was at the forefront of reform in the Anglican church before becoming a cardinal of the Roman Catholic Church. [[Sam Korn]] (smoddy) 20:52, 17 April 2009 (UTC)[reply]
The most famous case of converting to a completely different religion is Sabbatai Zev... AnonMoos (talk) 03:40, 18 April 2009 (UTC)[reply]

Media bias study about Vietnam

Hi. I was wondering if someone could help me find a bit of info I have been trying to get for a while.

I remember hearing a while ago about (what was described as) a famous study done during the Vietnam War. I think it was done at Columbia University, but don't hold me to that. It was about media bias, and consisted of showing the same clips of news coverage of the War to groups of pro- and anti-War students separately. They both thought that the coverage - the same clips - was biased against their cause.

I cannot for the life of me remember who did the study, and google searches have so far been fruitless. Does anyone know where I can find more info about the study? Thanks in advance, Batmanand | Talk 13:43, 17 April 2009 (UTC)[reply]

See hostile media effect, which cites original studies. --Sean 16:33, 17 April 2009 (UTC)[reply]
Yep that is it - brilliant! I'm sure there is a Vietnam Columbia study somewhere, but the references in the page are great. Thanks! Batmanand | Talk 20:22, 17 April 2009 (UTC)[reply]

American Obsesssion with tax

Hello Wikipedia,

The 'Tax tea parties' are seemingly everywhere in the States (at least, they're all over Huffington Post, which is all I have to go on). Anyway, I get that culturally, Americans are less tolerant of tax than say, us brits, but I mean,seriously! No one likes paying tax but why do you guys loathe it so much? Wouldn't you like less crime, cleaner air etc? Is there something i'm just not getting, like a scandal of some sort that means the american government is forever percieved as untrustworthy? I'm not looking for a debate (although it sure would be fun!), but just some reasons why americans are thus. Thanks212.183.134.209 (talk) ā€”Preceding undated comment added 15:05, 17 April 2009 (UTC).[reply]

Because the American Revolution was essentially a revolt against taxes...the Intolerable Acts and all that. Of course it was really against taxation without representation but does an average person actually know what that means? And do they know about all the other things that led to the Revolution? And do they know why they have taxes now and what their taxes pay for? Probably not. But their foundation myth includes a tax revolt and that's a good enough excuse. Adam Bishop (talk) 15:35, 17 April 2009 (UTC)[reply]
It's certainly a cultural thing in America and the recent bailouts have only fanned the flames. I think most people are OK with some tax, but many object to using so much tax money to prop up failing companies. At any rate, the number from "Tea Party" officials (or whatever) were slightly over 200,000. Even if you multiply that out to a few million who wanted to protest but were unable to... you end up with a very small fraction of the American people. Tomdobb (talk) 16:20, 17 April 2009 (UTC)[reply]
A main goal was to promote tax cuts or extention of existing tax cuts for the wealthy. The "Tea Parties" were a publicity effort organized and promoted and funded by right wing media, a few wealthy individuals, and right wing political groups, with additional attendance by Libertarians and people hurting from the economic downturn. In fact, Gallup polls show that Americans are satisfied with present tax rates. 61% think the income tax is "fair" (more than any result reported back to 1946, save for 2003-2005) and 35% say "unfair." Respondants said that "Lower income people," and Middle income people" are paying their "fair share" but overwhelmingly said "Upper income people " are paying "too little." Edison (talk) 16:31, 17 April 2009 (UTC)[reply]
My understanding is that the people are protesting the government spending in that they expect their taxes to increase in the next few years. The taxation without representation argument appears to stem from the fact that the majority of protesters appeared to be conservatives who do not have enough seats in congress to do anything. Or it could be that they don't feel that politicians are representative of the people (how many senators would pass as a regular guy on the street?) 65.121.141.34 (talk) 16:42, 17 April 2009 (UTC)[reply]
Most Americans are angry about billions in bailouts give to Wall Street financial "experts" who created the present economic crisis, and then feel they are entitled to millions in "bonuses" for all they accomplished, or as "retention bonuses" so they don't quit and take a nonexistent job at another firm. These greedy financial geniuses are probably as popular with the average taxp[ayer as Marie Antoinette and her hubby were with Parisians circa 1793. This outrage is somehow supposed to represent a demand for continued tax breaks for the wealthy. Edison (talk) 16:49, 17 April 2009 (UTC)[reply]
Another factor is the different view of government in the US versus Europe. There is a traditional opposition toward "Big Government" in the US, with preference for free market capitalism, infused with themes of self sufficiency and rugged individualism, with allusions to the wild west cowboy mystique. The European model, where government is much more involved in providing things for its citizens is decried by US conservatives (the ones who are holding the Tea Parties) as socialism and a welfare state. In their minds, individuals know how to use their money best, so people should keep their own money, instead of being forced to give it to the government. In the mind of the conservatives, government messes up anything it touches (with the notable exception of military defense), so the less the government is involved, the better - less taxes means that the government does less. That said, there is a view by some that the recent tea parties aren't representative of actual opinion, but are rather an astroturfing stunt orchestrated by the Republican party, trying to discredit their opponents the Democrats, and gain more support after the Republicans' recent electoral defeat. -- 128.104.112.117 (talk) 19:24, 17 April 2009 (UTC)[reply]
Heh. I was gonna say that, but you beat me to it. Ā :) Who then was a gentleman? (talk) 04:03, 18 April 2009 (UTC)[reply]

I pay taxes so the rich don't have to. DOR (HK) (talk) 22:19, 17 April 2009 (UTC)[reply]


Those were some really great answers -thanks guys! ā€”Preceding unsigned comment added by 212.183.134.208 (talk) 14:21, 18 April 2009 (UTC)[reply]

Material

What is 'Action Leather'?96.53.149.117 (talk) 16:32, 17 April 2009 (UTC)[reply]

speeding ticket myth?

I have heard that if you get a speeding ticket, what you do is send in a check for a trivial amount more then you owe (I heard 10 cents). If you do that, they will not cash the check and you in essence will have gotten of without paying. This sound preposterous to me, but then there are stranger things in heaven and on earth right? Does anyone know if there is truth to this? 65.121.141.34 (talk) 16:37, 17 April 2009 (UTC)[reply]

It's implausible sounding.. and it's a well-known myth, see snopes for more. Friday (talk) 16:43, 17 April 2009 (UTC)[reply]
But if you carefully check all the details on the ticket, you might get lucky and find an error. I once got off a parking fine because the hand-written ticket said "Wednesday, <date> <year>", when the actual day of the week was something else. I made the point in my letter that the last time that date had been a Wednesday was something like 5 years earlier, when I could prove I lived 1000 km away and did not own the car in question. I never heard back from the authorities. -- JackofOz (talk) 20:46, 17 April 2009 (UTC)[reply]
All your pedant are belong to us! BrainyBabe (talk) 23:43, 17 April 2009 (UTC)[reply]

Functional form of von-Neumann Morgenstern utility function - any help much appreciatedĀ !!!

Just realised that humanties harbours economics so apologies for also posting this in the maths section.

For my studies i need to use the von-Neumann Morgenstern utility function but i am not aware of its function form (i.e. a cobb douglas is Y=K^aL^b for example). I am not trying to get the answer to save me the work of doing so i am just trying to do background research.

Thank you very much for your time ā€”Preceding unsigned comment added by 86.163.232.191 (talk) 16:59, 17 April 2009 (UTC)[reply]

Are you sure it has a specific functional form? From my extremely short check on google, it looks like the von Neumannā€“Morgenstern utility functions are a class of functions that satisfy certain axioms regarding risk aversion. Zain Ebrahim (talk) 23:51, 17 April 2009 (UTC)[reply]
vNM utility (also called expected utility) has the form that expected utility is the sum of the utility from each possible event times the probability that that event occurs. (These events must form a partition in state space). So if there is one event that occurs with probability 0.8 and gives utility 1, and another that occurs with probability 0.2 and gives utility 2, then EU=(0.8*1)+(0.2*2)=1.2. There are axiomatic foundations for expected utility as Zain Ebrahim says, but these make no assumptions on risk aversion.79.70.246.6 (talk) 10:00, 18 April 2009 (UTC)[reply]

Parents' Relations

My parents share a common great-grandmother. She married twice and had my mother's grandmother with one husband and my father's grandmother with another husband. What relation are my parents to each other (besides the obvious husband-wife)? What interesting inter-relationships does this create between my siblings and me and my parents and me? Also, is this common (in modern times) or would an occurrence like this be somewhat rare? 157.127.124.14 (talk) 17:03, 17 April 2009 (UTC)[reply]

According to Cousin#Half cousins, they're half second cousins because their grandmothers are half siblings. That would make you your own half third cousin. Cool! Zain Ebrahim (talk) 17:20, 17 April 2009 (UTC)[reply]
Oops, I totally miscounted. They actually share a common great-great grandmother. I suppose that makes them half third cousins and makes me my own half fourth cousin.Ā :) Thanks! 157.127.124.14 (talk) 17:33, 17 April 2009 (UTC)[reply]

Nuns

Which nuns do never speak? I remember I heard something about nuns who live in silence all the time. Do you know what are they called? ā€”Preceding unsigned comment added by Atacamadesert12 (talk ā€¢ contribs) 17:18, 17 April 2009 (UTC)[reply]

Just guessing here, but Cistercian (Trappists in particular) monks and nuns are permitted to speak under certain specific strict conditions - but they do not take vows of silence. Were you thinking of them? Zain Ebrahim (talk) 17:37, 17 April 2009 (UTC)[reply]
We do have an article on them. Unfortunately, it has remained to be a red link as none of the nuns ever broke their silence. The entry - well there was none - was eventually deleted for being not not able. We also have an article on the Trappist Family Singers in Braille in stereophonic silence. --Cookatoo.ergo.ZooM (talk) 19:16, 17 April 2009 (UTC)[reply]
Thank you thank you. Order the veal, tip your waitresses! Remember folks, the 11:00 show is completely different from the 7:00 show! Good night all! --Jayron32.talk.contribs 19:22, 17 April 2009 (UTC)[reply]
Many Catholic contemplative orders (e.g., the Poor Clares) observe long periods of silence, though not "all the time." It'd be difficult for a sister to confess her sins, for example, without speaking. --- OtherDave (talk) 20:22, 17 April 2009 (UTC)[reply]

Nationalities on Titanic?

I know most of them (including crew) were British... but do you have numbers? --201.254.83.172 (talk) 20:33, 17 April 2009 (UTC)[reply]

Found this great page. 334 British and 306 Americans. It breaks it down by class too: First class 212 American, far dominant. A lot more British in second class with 169. Third class was close: 120 Brits, 113 Irish, 104 Swedes. Interesting: one Mexican - a first class passenger. Grsz11 20:39, 17 April 2009 (UTC)[reply]
Don. Manuel E. Uruchurtu meltBanana 22:44, 17 April 2009 (UTC)[reply]

FTSE 100 Index - recent history of

What and when was the highest ever value of the FTSE 100 Index, and what and when was its lowest value since then? (It has recently gone above 4000 again - things are looking up). 78.149.114.51 (talk) 20:49, 17 April 2009 (UTC)[reply]

It peaked at 6930.2 on 30 Dec 1999.[30] ā€”D. Monack talk 01:22, 18 April 2009 (UTC)[reply]
I haven't looked into it, that looks like the highest value of the index, but due to the way its worked out, that might not actually be the highest value of the shares in it, but that would be very complicated, because occasionally individual company's shares go into and leave the top 100. Here might help generally, (if you have good eyes!).Grandiose2 (me,talk,contribs) 14:02, 18 April 2009 (UTC)[reply]

Ethics of Parenting

I was wondering why and how do we have the right to chose to bring somebody to life. By definition we impose the state of life to our children. Life can be good but can also be hard, or even unbearable. Is this ethical? Since we do not know how our childrens lives will develop, if they are gonna be born with congenital problems, if they will suffer from leukemia at their 3 and so ever, no matter how hard we try avoid anything bad from happening to them. Sure some of the mishaps in life will result from THEIR faults and they will be THEIR responsibility, some will still be just "bad luck", and will happen. If we make children and everything goes normal, no problem (under the hypothesis that "normal" means more good than bad things as our children perceive them, or at least in equal amount). But if we make a child and gets afflicted with misfortunes she is not willing to endure, then she can blame us because we sort of bet on her behalf by bringing her to the game called life... Some examples would be impoverished economies, hereditary disorders, periods of war or instability etc. But it can be generalized to almost all aspects of life. Since humans are the only species having full consciousness about themselves and their surroundings, and really know about the facts of reproduction while have effective means of contraception, is it ethical to make children? Based on which criteria? Why the fact that *I* "feel ready to be a parent" should affect someone on the most fundamental basis, his/her _existence_? And if I have that right, why can't I take that life -or any other life, since they could be my children- "back"? How can I have the right to chose to bring somebody to life, when that decision has to do with his/her very existence? And since I do have the right obviously, why shouldn't I kill somebody? It's just the reverse case, but the degree of influence over one's life is still the same! It only would be reasonable if we, a priori, take life as a good thing in general, but we all know that this is not always the case. Of course a negative answer on the ethical matter would have dire consequences for the survival of the human race, but I think it is a very fundamental topic because we do not discuss here the quality/meaning of life but it's existence itself. I am really interested in hearing your opinion on that and your suggestions of any relevant bibliography on the issue. The important question remains: "How ethical is to chose to bring someone to life?" Makischa (talk) 22:09, 17 April 2009 (UTC)[reply]

By that logic, doing anything that might result in a "bad" outcome for someone else is not ethical. So I shouldn't drive to work today because some moron on a cell phone might knock me and kill himself thus depriving his family of a breadwinner. Zain Ebrahim (talk) 22:40, 17 April 2009 (UTC)[reply]
That "moron" CHOSE to be talking to his cell, so he has to deal with the consequenses of HIS action. Our children do not have the choice of being born. That's the problem here.Makischa (talk) 22:58, 17 April 2009 (UTC)[reply]
And what did his children do? Zain Ebrahim (talk) 23:00, 17 April 2009 (UTC)[reply]
Nothing! He also chose to have them, and decided to behave stupidly. This is one example, as the ones I wrote above. They came to a world having an irresponsible father who would make them orphans and won't have the opportunity to go to college becasue will have to work for their subsistence! That's what it's all about. ā€”Preceding unsigned comment added by Makischa (talk ā€¢ contribs) 23:04, 17 April 2009 (UTC)[reply]
I'm not talking about his unethical choice to have children - I'm talking about my unethical choice to go to work. Zain Ebrahim (talk) 23:05, 17 April 2009 (UTC)[reply]
You are part of this world, and in this case you are a part of the "bad things" which are inherently associated with life, since we cannot control everything. So he brought his children to a world where these things can happen. And they actually do happen. It wasn't obviously unethical of you to go to work.Makischa (talk) 23:09, 17 April 2009 (UTC)[reply]
If having children is unethical (because bad things might happen to them) then driving to work is also unethical. Zain Ebrahim (talk) 23:14, 17 April 2009 (UTC)[reply]
I am wondering if bringing children to a world where bad things might happen is ethical or not. You, or anyone, can do whatever he wants, and your actions might consist a part of the lack of ethics we have in our world along with the misfortunes that are nobody's fault. This is our world and that's a fact. But have we the right to bring someone to such a world? Makischa (talk) 23:21, 17 April 2009 (UTC)[reply]
Well, do I have the right to go to work? Zain Ebrahim (talk) 23:28, 17 April 2009 (UTC)[reply]
As I said you can do whatever you want, ethical or not. It's up to you. And actually we don't care because even if a miracle happened and everyone started to behave ethically, misfortunes attributed to chance alone will continue to happen. Makischa (talk) 23:31, 17 April 2009 (UTC)[reply]

Driving someone to work does not guarantee death for them. Coming into this world, however, does guarantee death. When you have a child, there is no doubt that that child will die someday. Wrad (talk) 23:16, 17 April 2009 (UTC)[reply]

How is that relevant? Zain Ebrahim (talk) 23:19, 17 April 2009 (UTC)[reply]
How is it not? This is not something that might happen to your child, a major part of your arguments above. It is a guarantee. If you have a kid, bad things will definitely happen to him or her. There is no might or maybe. Wrad (talk) 23:22, 17 April 2009 (UTC)[reply]
So it is unethical to procreate because life is finite? Zain Ebrahim (talk) 23:28, 17 April 2009 (UTC)[reply]
Let's get rid of the fact of our childrens death since as you say it is certain. Let's assume that living a normal life as defined in my first post is enough for making our decision to have children ethical.Makischa (talk) 23:29, 17 April 2009 (UTC)[reply]
But good things will happen to them too. Isn't it said: "It is better to have loved and lost, than never to have loved at all"? - Then we must consider those moral/ethical systems where denying someone a benefit through inaction is considered as bad as actively causing them harm. Does the moral good that comes from potentially sparing your future child from debilitating cancer outweigh the moral harm of denying them the happiness of their wedding day or the simple joy of summer afternoons? -- 128.104.112.117 (talk) 23:59, 17 April 2009 (UTC)[reply]
I'm not denying them anything since they do not exist. If that was the case we should have as many children as biologically possible, because if you chose to have let's say 4, then you are denying your 5th, 7th, 20th child the happiness of their wedding day. The case your are considering, means inaction=harm and I think it is correct sometimes. But you cannot harm somebody who does not yet exist.Makischa (talk) 00:08, 18 April 2009 (UTC)[reply]

This has all the earmarks of a debate, where Makischa appears to be more interested in challenging every statement than in an answer. If there is a specific answer (and I certainly don't have one) we could reference it. If not, perhaps this should be moved off the Ref Desk to a forum for such matters. // BL \\ (talk) 00:05, 18 April 2009 (UTC)[reply]

You are right. I will be waiting for an expert opinion or reference. I think I've made my case clear now. Thank you.Makischa (talk) 00:08, 18 April 2009 (UTC)[reply]
Nobody on here is an expert; we're just fellow editors. But here's a thought to consider- is not allowing potential children to live unethical? Sure, they will never die, but they will also never get to live. Is withholding life from someone also unethical? I don't really agree with your original logic, but if you must take that point of view, you must consider the other side of it. --Alinnisawest,Dalek Empress (extermination requests here) 22:10, 18 April 2009 (UTC)[reply]

For one possible philosophy designed to quell all qualms such as User:Makischa apparently feels, read the classic satirical novel Erewhon by Samuel Butler... AnonMoos (talk) 03:46, 18 April 2009 (UTC)[reply]

I think one should only have children if you believe that they will probably be happy. I have no children, and will not have any in the future. Children are not pets or toys. 78.149.207.226 (talk) 12:24, 18 April 2009 (UTC)[reply]

April 18

Which crowns are these with Richard III and family?

The writing is difficult to decipher. Thanks! Catterick (talk) 04:54, 18 April 2009 (UTC)[reply]

Do you mean the 6 surrounding the King? If so, we have, from top left, Seyne (Saint?) Edward,(maybe that of Edward the Confessor who was King of England) England, France, Wales, (this one I can't understand), and Ireland. --TammyMoet (talk) 08:12, 18 April 2009 (UTC)[reply]
The symbol on the unknown--is that the Scottish thistle? If so, I'd still like to make out the exact lettering. Catterick (talk) 08:26, 18 April 2009 (UTC)[reply]
The cross fits with Edward the Confessor with similar arms (although they were made posthumously). - Jarry1250 (t, c) 10:13, 18 April 2009 (UTC)[reply]

I found a different version of this image that is much more legible. Actually the color version you posted here looks to me like a later copy of the black-and-white one, made by someone who didn't really know what they're copying, hence the illegible inscriptions. Anyway, the bottom right crest, which looks more like grapes than a thistle, is labeled "Gascoyn & Gyan", that is Gascony and Guyenne (Aquitaine). ā€” Kpalion(talk) 13:56, 18 April 2009 (UTC)[reply]

assignment help needed

I needed some information regarding my assignment. The subject is reagrding commercial law. And the question is based on contracts law ans sale of goods law. The question is as follows:

Brandy lives in the Fiji islands. She is a manufacturer of traditional tapa mats. This particular type of mats is known as ā€œmasiā€. Brandy sells mats in various colours with the traditional tapa mats or masi being a dark brown colour with dyed woolen edges. After successfully doing business for a few years within the tourism market Brandy decides to expand within the export markets. She subsequently establishes a website advertising the products for sale to overseas markets.

Sima in New Zealand operates a large cultural store where all types of traditional hand made items are being sold. One day while Sima is online she finds Brandyā€™s website. Knowing of the demand for traditional tapa mats within the Maori community she decides that it would be a good idea to import some of Brandyā€™s stock for sale in her shop. She particularly likes the variety of bright colours that are being used in the dyed woolen edges. Brandy sends Sima a contract to sign which includes the following provisions: 1. goods ā€“ 100 traditional tapa mats with coloured woolen edges 2. purchase price - $250 per mat 3. Contract terms ā€“ in accordance with Sale of Goods Act Fiji 4. shipment terms ā€“ CIF 5. delivery date ā€“ 90 days from entry into this contract 6. delivery port ā€“ Auckland Before Sima signs the contract she asks if Brandy can deliver the mats for the same price to Christchurch, and states that she wants half of the order to be traditional mats with dark brown woolen edges and half to have coloured woolen edges. Brandy states that there would be an extra charge of $25 for delivery to Christchurch, however, costs would be less for the 50 mats with only dark brown woolen edges. Sima then amends the contract purchase price per mat to ā€œ$262.50ā€, makes no other changes and signs the contract. In order to secure payment, Brandy is provided with an irrevocable letter of credit from Simaā€™s bank ā€“ Money Bank. In arranging the letter of credit Sima has provided Money Bank with a copy of the contract. Under the Letter of Credit Brandy is required to present the contract transportation documents, clear bill of lading, and evidence of premium insurance coverage before Money Bank will pay. The mats are delivered to the ship at Suva, and a Bill of Lading is issued for ā€œ100 traditional matsā€. The sea trip is good and due to fine weather the ship reaches Christchurch a day before the scheduled date. Sima however, has failed to make arrangements for early delivery and the crate containing the mats is sitting on the dock for one night. Unfortunately there is a snow storm on that particular night. When Sima finally collects the mats about 25 of the brown edged mats are damaged, and the other 50 tapa mats with colourful woolen edges have lost their bright colours and would need replacement with new colourful edges. Sima is furious and immediately rings Money Bank from her mobile phone. She gets even more furious when she is told that Brandy has been paid even though the bank did not collect the importation documents. Sima then attempts to claim the losses on her insurance. She discovers that the insurance policy is not a premium policy, but a standard policy, and does not extend cover to the mats after delivery.

Advise Sima of her rights and liabilities against the following: a) Brandy b) Money Bank ā€”Preceding unsigned comment added by Nalini DK (talk ā€¢ contribs) 07:36, 18 April 2009 (UTC)[reply]

You haven't said what the information is that you want. We are not going to do your assignment. (From the top of the page: If your question is homework, show that you have attempted an answer first, and we will try to help you past the stuck point. If you don't show an effort, you probably won't get help. The reference desk will not do your homework for you.)
You might get some useful information from Commercial law, but probably more relevantly from your course textbooks. Incidentally, you have not even said which legal system you are working in: I guess it is New Zealand, but if anybody is to help you, you need to make this clear. --ColinFine (talk) 09:21, 18 April 2009 (UTC)[reply]
See 2009 Fijian constitutional crisis - Fiji is in the middle of its 4 and a half coup of recent years and presently has no judges but plenty of police and soldiers. So the answer is...how well does Simi know the Prime Minister of Fiji? Rmhermen (talk) 05:13, 19 April 2009 (UTC)[reply]

Genesis - Creation

Have creationists considered the fact that whoever wrote Genesis (as well as other people) probably would not have been able to understand concepts such as cells, genes, etc? Vltava 68 07:38, 18 April 2009 (UTC)[reply]

Have you considered the fact that modern creationists are probably unable to understand concepts such as cells, genes, etc. either? ā€” Kpalion(talk) 12:12, 18 April 2009 (UTC)[reply]
Well...that's not true. Lots of "real" scientists are creationists. Do you think Michael Behe doesn't understand those concepts? Also, if the author of Genesis was God, then of course he would have understood cells and genes. Adam Bishop (talk) 13:35, 18 April 2009 (UTC)[reply]
Adam Bishop, given the article to which you refer, it is hard for most of us to accept that the individual named does understand cells and genes in the same way that almost all accredited scientists understand them. To go back to Kpalion Vltava's question, I would direct him/her to the various articles on Creationism, Intelligent design, Book of Genesis and their follow-up reading for information on what creationists might have considered. // BL \\ (talk) 15:32, 18 April 2009 (UTC)[reply]
Well, he probably knows more about cells and genes than I do! Adam Bishop (talk) 16:34, 18 April 2009 (UTC)[reply]
Definition of "expert": Someone who knows more about a subject than I do. Ā :) -- JackofOz (talk) 21:12, 18 April 2009 (UTC)[reply]
AB's second point still stands, though- if God was the true writer of Genesis, then obviously he'd understand genes, etc. So if creation (and divine inspiration) are correct, there's no problem. And if they aren't, then there's no problem either. So regardless of the answer (I'm presuming the answer to the OP's question is "yes", by the way, as most creationists likely aren't crackpots... just the ones you hear about all the time are. Sane people don't make for good news), it doesn't really matter. --Alinnisawest,Dalek Empress (extermination requests here) 22:06, 18 April 2009 (UTC)[reply]
There are those who think all creationists are crackpots, by definition; however, even if that opinion were true, it would have no bearing on what creationists might have considered in coming to their conclusions. The answer will more likely be "yes' for some, "no" for others and "who knows what they thought about?" for the rest. // BL \\ (talk) 22:16, 18 April 2009 (UTC)[reply]
An Answers in Genesis-style answer would be, "yes, of course the humans who wrote these things out originally didn't know about modern scientific concepts. However their words were guided by the hand of God, who understands them better than even our clunky human metaphors do. Why would you assume that it matters in this case? Whether or not the people who wrote down Genesis originally understood modern cell biology hardly affects whether their overall scheme for the history of creation was true or not." etc. etc. I'm not a Creationist but I don't find the question of who knew about cells to have any real importance to discussing its validity. --98.217.14.211 (talk) 03:14, 19 April 2009 (UTC)[reply]

What are the chances of a nuclear war in the next 100 years?

How have future historians projected it?--Whargarbl (talk) 12:06, 18 April 2009 (UTC)[reply]

See Doomsday Clock. ā€” Kpalion(talk) 12:20, 18 April 2009 (UTC)[reply]
Sorry, our nuclear-powered WP:CRYSTAL ball is glowing just a bit too brightly for me to see the answer. (P.S. What's a future historian?) Clarityfiend (talk) 00:17, 19 April 2009 (UTC)[reply]
The only time I've ever heard the phrase "future historian" is to refer to a hypothetical historian who is literally in the future, looking back and studying us. User:Whargarbl is probably thinking of a Futurist.
Seriously, nobody reputable is going to venture a guess that far ahead. Humans could be extinct or god-like by then. Clarityfiend (talk) 00:32, 19 April 2009 (UTC)[reply]
Yeah. About the best we can say is that the odds are less than 50%: humanity surviving for 100 years has been given 50% odds by a few authorities, and 50% is (following Bayesian orthodoxy) the position of maximum ignorance regarding that issue. Humanity could be extinguished by any number of disasters, so to avoid the conjunction fallacy, we get <50%. (I assume that extinction and nuclear war are related; but even if they're independent variables, humanity has to exist for nuclear war to happen, so the odds are still necessarily reduced.) --Gwern (contribs) 02:47 19 April 2009 (GMT)

Why is there no article Islam and communism?

Surely the links between these two topics are worthy of exploration?--Whargarbl (talk) 12:41, 18 April 2009 (UTC)[reply]

Maybe you're the first person to have got around to thinking about it. There has to be a first; and we're still - after all this time - creating hundreds of new articles on significant topics every day of the week, many of which would cause various people to be surprised that we didn't already have something on that. The thing is, it's not like a bunch of terribly self-important people sit around and decide which articles should be written and which not. No, the world community decides, and in particular, the decision to commence any one article is a decision by a single person - you. There are checks and balances, of course, and all articles are subject to scrutiny and challenge in regard to notability. So, the opportunity is now available to you - or anyone reading this - to create the article. Best wishes, and I look forward to reading it. -- JackofOz (talk) 12:56, 18 April 2009 (UTC)[reply]
Or you could start by expanding this very short section: Communism and religion #Communism and Islam. ā€” Kpalion(talk) 13:12, 18 April 2009 (UTC)[reply]
Indeed, one could also explore the relationship between Communist states and Islamic ones as well. The Suez Crisis may be of particular interest to one writing such an article. Also, the article essentially already exists at National communism. Read it and see... --Jayron32.talk.contribs 20:09, 18 April 2009 (UTC)[reply]
Islamic socialism might also be useful. [[Sam Korn]] (smoddy) 11:27, 19 April 2009 (UTC)[reply]

Status of post-War German states

Whta levels of autonomy did German states have while the future of post-World-War-II Germany was being discussed. Our article on the coat of arms of Lower Saxony has these 'dubious' statements:

  • After World War II, the province of Hanover became an independent state
  • Brunswick, which was an independent state

and I'm wondering here whether the confusion here is simply over the use of the word state as a translation of land (the German word) rather than Sovereign state. Though, German 'provinces' (Laender) have always had more autonomy. Could someone clarify the position here (anyone who knows should edit the article really)? Grandiose2 (me,talk,contribs) 13:15, 18 April 2009 (UTC)[reply]

I don't doubt those statements at face value, given Germany's history as a nation. The fact that there would be one German state that was uniquely sovereign over all German lands has really only existed since the 20th century. Various independant states existed throught Germany in all of history; it is not surprising that in the chaos after World War II, any of a number of short-lived Independent states did not "spring up" after the war. Even during the so-called "German Empire" period, many constituent parts of the Empire, such as the Kingdom of Bavaria, operated essentially independently from the Prussian-dominated empure, and were de-facto independent states in all but name. To answer the question about Hanover specifically, there had been a Kingdom of Hanover from 1814-1866 (which was in personal union with the Kingdom of Great Britain until 1837). AFter 1866, it was administered from Prussia as the Province of Hanover and was only formally integrated into the new Federal West Germany in 1946/47. --Jayron32.talk.contribs 20:07, 18 April 2009 (UTC)[reply]

April 19

Website for finding theme (literature)

Is there any website that allow to find a theme for any stories? 66.75.241.44 (talk) 03:35, 19 April 2009 (UTC)[reply]

Insofar as theme is a part of literary criticism, it is largely up to the subjective view of the reader as to the theme of a book or story. I think it may be hard to determine an objective way to categorize such an idea reliably, and I don't know that any website makes any attempt to do so. Your question is also a little vague. Could you expand on it? To what end are you looking for this information. --Jayron32.talk.contribs 04:43, 19 April 2009 (UTC)[reply]

Best way to stop a prima donna behaving as such?

What is the best way to stop a prima donna (in the metaphorical sense) from behaving as such? Thanks. 89.242.147.172 (talk) 10:54, 19 April 2009 (UTC)[reply]

Acronyms in American legislation

Why do American legislators insist on giving their bills ridiculous acronyms? Looking through a couple of the relevant categories, I find USA PATRIOT, PROTECT, RAVE, BALANCE, CARE, CALM, COPE, REAL, SAFE (twice!), SKILLS, and USA, and I could find many more if I looked through THOMAS. Is there some historical reason? Are there any other countries with similarly absurd practices? --superioridad (discusiĆ³n) 11:03, 19 April 2009 (UTC)[reply]